[obm-l] OBM 2022 Problema

2023-06-13 Por tôpico Prof. Douglas Oliveira
Determine o maior inteiro positivo k para o qual a afirmação é verdadeira:
Dados k subconjuntos distintos do conjunto {1, 2, 3, ..., 2023}, cada um
com 1011 elementos, é possível particionar os subconjuntos em duas coleções
em  a forma que quaisquer dois subconjuntos na mesma coleção têm algum
elemento em comum.



Abraço do Douglas Oliveira.

-- 
Esta mensagem foi verificada pelo sistema de antiv�rus e
 acredita-se estar livre de perigo.



[obm-l] Conjuntos

2022-11-18 Por tôpico Prof. Douglas Oliveira
Olá amigos mestres, podem me indicar bons livros de conjuntos, que trabalham
com álgebra dos conjuntos de todas as formas possíveis, por exemplo:
Trabalham com desigualdade de Bon Ferroni, mapas de Karnaugh,
relações com 4 conjuntos e etc.

Att
Prof Douglas Oliveira

-- 
Esta mensagem foi verificada pelo sistema de antiv�rus e
 acredita-se estar livre de perigo.



[obm-l] Re: [obm-l] Dúvida

2021-11-16 Por tôpico Prof. Douglas Oliveira
Equação de Pell

Em seg., 15 de nov. de 2021 13:36, Pedro José 
escreveu:

> Boa tarde!
>
> Alguém saberia como resolver a seguinte equação:
>
> x^2-7y^2=1, x,y em Z?
>
> Fiz a-7b=1 e achei a= 8 +7k e b=1 +K
> Logo fica fácil que para k=-1 funciona x^2=1 e y^2=0.
> Também funciona para k=8 x^2=64 e y^2=9.
> Mas não sei nem como achar mais soluções nem como provar que só são essas.
> Alguém poderia me dar uma orientação?
>
> Cordialmente,
> PJMS
>
> --
> Esta mensagem foi verificada pelo sistema de antivírus e
> acredita-se estar livre de perigo.

-- 
Esta mensagem foi verificada pelo sistema de antiv�rus e
 acredita-se estar livre de perigo.



[obm-l] Ajuda numa questão da OBM 1987

2021-07-20 Por tôpico Prof. Douglas Oliveira
*Tem-se um bolo em forma de prisma triangular, cuja base está em um plano
horizontal. Dois indivíduos vão dividir o bolo de acordo com a seguinte
regra: o primeiro escolhe um ponto na base superior do bolo e o segundo
corta o bolo por um plano vertical à sua escolha, passando porém pelo ponto
escolhido, e seleciona para si um dos pedaços em que dividiu o bolo.
Qual deve ser a estratégia para o primeiro e qual deve ser a fração do
volume do bolo que ele espera obter?*

*Abraço do Douglas.*

-- 
Esta mensagem foi verificada pelo sistema de antiv�rus e
 acredita-se estar livre de perigo.



[obm-l] Re: [obm-l] construção geométrica

2020-09-21 Por tôpico Prof. Douglas Oliveira
Olá Luís, rabisquei aqui no papel agora, e pensei assim...

Vamos considerar primeiro o triângulo ABC inscrito no círculo, onde AB=c,
AC=b e BC=a.
Desta forma vamos considerar o problema de "ponta cabeça", onde P se
encontra no círculo e que PA=x e PC=y,
logo PC=x+y.
Vou numerar os passos para  fim de organização.

1) Aplicando ptolomeu no quadrilátero ABCP teremos a razão entre os
segmentos x e y, logo x/y=(c-b)/(b-a).

2) Agora divida o segmento AC nesta razão dada utilizando régua e compasso,
e chame esse ponto de N pertencente à AC

3) Encontre o conjugado hamônico do ponto N fazendo a construção de um
quadrilátero completo assim vai encontrar na reta
 suporte AC o ponto M (conjugado harmonico de N)

4) MN é o diâmetro do círculo de apolonius, agora basta desenhar este
círculo e o ponto de interseção dele
com o circulo original é o ponto que você procura

Bom acho que é isso.
Se errei em alguma coisa, por favor me corrija
Grande abraço
Douglas Oliveira (RCMAT)

Em qua., 10 de jun. de 2020 às 17:24, Luís Lopes 
escreveu:

> Sauda,c~oes,
>
> Recebi o seguinte problema:
>
> Construir P no circuncírculo de um triângulo ABC dado
> tal que PA+PB=PC.
>
> Alguém saberia fazer ?
>
> Obrigado.
>
> Abs,
> Luís
>
>
>
> --
> Esta mensagem foi verificada pelo sistema de antivírus e
> acredita-se estar livre de perigo.
>

-- 
Esta mensagem foi verificada pelo sistema de antiv�rus e
 acredita-se estar livre de perigo.



[obm-l] Re: [obm-l] Re: [obm-l] Re: [obm-l] Ajuda em teoria dos números

2020-07-24 Por tôpico Prof. Douglas Oliveira
Obrigado Claudio e Esdras, fatoração show


Em sex., 24 de jul. de 2020 às 11:12, Esdras Muniz <
esdrasmunizm...@gmail.com> escreveu:

> Se for solução inteira positiva, acho que só tem 3 e 4. Vc supõe spdg x
> maior ou igual a y, vê que y=1 não tem solução e x=y tb não. Daí, x>y>1.
> Fatorando a expressão, fica: (xy-8-(x-y))(xy-8+(x-y))=15. Como
> (xy-8-(x-y))>(xy-8+(x-y))>-2. Temos que ou (xy-8-(x-y))=1 e (xy-8+(x-y))=15,
> o que não tem soluções inteiras positivas, ou (xy-8-(x-y))=3 e (xy-8+(x-y))=5,
> cujas únicas soluções inteiras são x=4 e y=3.
>
> Em sex, 24 de jul de 2020 10:36, Claudio Buffara <
> claudio.buff...@gmail.com> escreveu:
>
>> Pelo que entendi, a solução é a porção dessa curva algébrica situada no
>> 1o quadrante.
>> Dá pra fazer isso no Wolfram Alpha, com o comando plot (x*y-7)^2 - x^2 -
>> y^2 = 0.
>>
>> []s,
>> Claudio.
>>
>> On Fri, Jul 24, 2020 at 9:58 AM Prof. Douglas Oliveira <
>> profdouglaso.del...@gmail.com> wrote:
>>
>>> Preciso de ajuda para encontrar todas as soluções não negativas da
>>> equação
>>> (xy-7)^2=x^2+y^2.
>>>
>>> Desde já agradeço a ajuda
>>> Douglas Oliveira
>>>
>>> --
>>> Esta mensagem foi verificada pelo sistema de antivírus e
>>> acredita-se estar livre de perigo.
>>
>>
>> --
>> Esta mensagem foi verificada pelo sistema de antivírus e
>> acredita-se estar livre de perigo.
>
>
> --
> Esta mensagem foi verificada pelo sistema de antivírus e
> acredita-se estar livre de perigo.

-- 
Esta mensagem foi verificada pelo sistema de antiv�rus e
 acredita-se estar livre de perigo.



[obm-l] Ajuda em teoria dos números

2020-07-24 Por tôpico Prof. Douglas Oliveira
Preciso de ajuda para encontrar todas as soluções não negativas da equação
(xy-7)^2=x^2+y^2.

Desde já agradeço a ajuda
Douglas Oliveira

-- 
Esta mensagem foi verificada pelo sistema de antiv�rus e
 acredita-se estar livre de perigo.



[obm-l] Re: [obm-l] Re: [obm-l] Números complexos e equações

2020-06-18 Por tôpico Prof. Douglas Oliveira
Opa mestre Claudio, muito obrigado, gostei da solução.

Douglas Oliveira

Em qua, 17 de jun de 2020 17:00, Claudio Buffara 
escreveu:

> Aquele 1+i sugere que se forme uma equação em z, onde z = (1+i)/raiz(2) *
> x, ou seja, cujas raízes sejam as da equação original giradas de 45 graus
> no sentido anti-horário e sem coeficientes complexos.
> z = (1+i)/raiz(2) * x ==> x = (1-i)/raiz(2) * z
> Assim, x^4 + 4(1+i)x + 1 = 0 ==> -z^4 + 4*raiz(2)*z + 1 = 0
> Consideremos f(z) = z^4 - 4*raiz(2)*z - 1  (multiplicar os coeficientes
> por -1 não altera as raízes).
> f(-1) = 4*raiz(2) > 0
> f(0) = -1 < 0
> f(raiz(2)) = -5 < 0
> f(2) =15 - 8*raiz(2) > 0 ==> f tem (pelo menos) duas raízes reais: uma
> entre -1 e 0 e outra entre raiz(2) e 2.
> Mas f'(z) = 4z^3 - 4*raiz(2) ==> f'(z) < 0 para z < 2^(1/6) (logo, para z
> < 0) e f'(z) > 0 para z > 2^(1/6) (logo, para z > raiz(2)), de modo que
> estas são as únicas raízes reais de f.
> Se duas das raízes da equação original, ao serem giradas de 45 graus no
> sentido anti-horário, se tornam reais, então aquelas raízes estavam na reta
> Im(z) = -Re(z).
> Além disso, como, após giradas, uma se tornou negativa e a outra positiva,
> isso significa que a primeira está no 2o quadrante e a segunda no 4o
> quadrante.
>
> Dadas as magnitudes das raízes giradas (a primeira entre -1 e 0 e a
> segunda maior do que raiz(2)), também concluímos que a soma delas está no
> 4o quadrante, ou seja, é da forma p*(1-i), com p > 0.
> Além disso, o produto delas é da forma (1/q)*i, com q > 0.
>
> Chame as outras duas raízes da equação original de a e b.
> Então, como a soma das raízes é zero, vale a+b = -p(1-i) = p(-1+i): um
> ponto do 2o quadrante sobre a reta Im(z) = -Re(z)   (1)
> Como o produto das raízes é 1, vale a*b = -q*i, um ponto do eixo
> imaginário negativo
> A localização do produto a*b implica que a/|a| e b/|b| são números
> complexos (de módulo unitário) e simétricos em relação à reta Im(z) =
> -Re(z)   (2)
> (1) e (2) implicam que a e b têm o mesmo módulo R
>
> (2) também implica que, sobre a e b:
> OU ambos pertencem ao 2o quadrante
> OU um deles pertence ao 1o e o outro ao 3o quadrante
> OU ambos pertencem ao 4o quadrante.
>
> De cara dá pra eliminar a última alternativa, já que isso implicaria que
> a+b pertence ao 4o quadrante, o que não é o caso.
>
> Resta eliminar a 1a alternativa.
> Assim, suponhamos que a e b pertencem ao 2o quadrante.
>
> Neste caso, a+b = p(-1+i) ==> |a+b| = p*raiz(2) > R*raiz(2) ==> 2*p^2 >
> 2*R^2
> E também, de qualquer jeito, ab = -qi ==> q = R^2
>
> Da equação, também sabemos que ab + ac + ad + bc + bd + cd = 0 ==>
> ab + cd + (a+b)(c+d) = 0 ==>
> -q*i + (1/q)*i + p(-1+i)*p*(1-i) = 0 ==>
> 1/q - q + 2p^2 = 0
> 1/q - q + 2R^2 < 0 ==>
> 1/R^2 - R^2 + 2*R^2 < 1/R^2 + < 0 ==> contradição ==> a e b não pertencem
> ao 2o quadrante.
>
> Logo, temos que concluir que, sobre as outras duas raízes, que uma
> pertence ao 1o e a outra ao 3o quadrante.
>
> []s,
> Claudio.
>
>
> On Wed, Jun 17, 2020 at 9:01 AM Prof. Douglas Oliveira <
> profdouglaso.del...@gmail.com> wrote:
>
>> Olá, gostaria de uma ajuda para localizar as raízes da
>> equação x^4+4(1+i)x+1=0, saber em qual quadrante estão, joguei no MAPLE e
>> percebi que existe uma em cada quadrante.
>>
>> Mas não consigo achar uma saída.
>>
>> Obrigado.
>> Douglas Oliveira
>>
>> --
>> Esta mensagem foi verificada pelo sistema de antivírus e
>> acredita-se estar livre de perigo.
>
>
> --
> Esta mensagem foi verificada pelo sistema de antivírus e
> acredita-se estar livre de perigo.

-- 
Esta mensagem foi verificada pelo sistema de antiv�rus e
 acredita-se estar livre de perigo.



[obm-l] Números complexos e equações

2020-06-17 Por tôpico Prof. Douglas Oliveira
Olá, gostaria de uma ajuda para localizar as raízes da
equação x^4+4(1+i)x+1=0, saber em qual quadrante estão, joguei no MAPLE e
percebi que existe uma em cada quadrante.

Mas não consigo achar uma saída.

Obrigado.
Douglas Oliveira

-- 
Esta mensagem foi verificada pelo sistema de antiv�rus e
 acredita-se estar livre de perigo.



[obm-l] Ajuda em trigonometria

2020-04-29 Por tôpico Prof. Douglas Oliveira
Olá amigos, preciso de uma ajuda no seguinte problema abaixo:

Quero descobrir a solução geral para a equação trigonométrica

cos(ax+b)+cos(cx+d)=cos(ex+f)+cos(gx+h)


Sempre que nos deparamos com aqueles problemas de perseguição angular ou
outro tipo de problema de ângulos adventícios, geralmente caímos em um tipo
de equação desta.

Gostaria de uma ajuda, indicação de algum artigo, ou trabalho que fale sobe
isso. Pois acredito que já deve existir algo nesse sentido.

Desde já, muitíssimo obrigado.

Um grande abraço do
Douglas Oliveira

-- 
Esta mensagem foi verificada pelo sistema de antiv�rus e
 acredita-se estar livre de perigo.



Re: [obm-l] Dois problemas

2020-04-26 Por tôpico Prof. Douglas Oliveira
Hum , para o primeiro problema, acredito que deve existir alguma
sequencia periódica, tal que a_n+k=a_n,
ou seja, n(n+1)/2=(n+k)(n+k+1)/2 (mod10).
Logo 2nk+k^2+k=0 (mod20), fácil ver que k=20 satisfaz o problema, logo
a_n+20=a_n, para todo n.
Vamos calcular a_1+a_2+a_3+a_4+...a_20=70.
Acredito que para cada valor de n, podemos fazer o seguinte que n=20t+r,
onde r é o resto na divisão de n por 20.
Assim a soma a_1+a_2+a_3+a_4+...+a_n=tx70+a_1+a_2+a_3+...+a_r, desta forma,
fica dependendo do valor de n.
É isso.

Forte abraço
Douglas Oliveira

Em dom., 26 de abr. de 2020 às 19:35, Rogério Possi Júnior <
roposs...@hotmail.com> escreveu:

> Boa noite.
>
> Quem pode ajudar com esses dois problemas:
>
> 1) (Ibero-1992) Para cada inteiro positivo n, seja a_n o último dígito de
> 1+2+3+...+n. Calcule a_1+a_2+...+a_n.
>
> 2) (UK-1997) N é um número inteiro de 4 dígitos não terminado em zero, e
> R(N) é o número inteiro de 4 dígitos obtido pela reversão dos dígitos de N;
> por exemplo R(3275)=5723. Determine todos os inteiros N ára os quais
> R(N)=4N+3.
>
> Sds,
>
> Rogério
>
>
> --
> Esta mensagem foi verificada pelo sistema de antivírus e
> acredita-se estar livre de perigo.
>

-- 
Esta mensagem foi verificada pelo sistema de antiv�rus e
 acredita-se estar livre de perigo.



[obm-l] Re: [obm-l] Re: [obm-l] Combinatória

2020-04-06 Por tôpico Prof. Douglas Oliveira
Já foi respondido aqui na lista

https://www.mail-archive.com/obm-l@mat.puc-rio.br/msg50069.html

Eu e o Ralph.

Douglas Oliveira.
Um abraço.

Em seg, 6 de abr de 2020 19:53, Anderson Torres <
torres.anderson...@gmail.com> escreveu:

> Em qua., 11 de mar. de 2020 às 23:10, Vanderlei Nemitz
>  escreveu:
> >
> > Boa noite!
> > Alguém tem uma ideia para esse problema?
> >
> > Muito obrigado!
> >
> > De quantos modos se podem sentar em fila, 3 ingleses, 3 franceses e 3
> turcos, de modo que não fiquem dois compatriotas juntos?
> >
> >
> > A resposta é 37584.
> >
>
> Não me parece ser algo fácil.
>
> Minha ideia aqui seria simplesmente fazer inclusão-exclusão. Se
> calculássemos de quantas formas pelo menos um par de compatriotas
> acaba lado a lado, bastaria achar o complementar disso.
>
> Mas dá um trabalho...
>
> >
> > --
> > Esta mensagem foi verificada pelo sistema de antivírus e
> > acredita-se estar livre de perigo.
>
> --
> Esta mensagem foi verificada pelo sistema de antivírus e
>  acredita-se estar livre de perigo.
>
>
> =
> Instru�ões para entrar na lista, sair da lista e usar a lista em
> http://www.mat.puc-rio.br/~obmlistas/obm-l.html
> =
>

-- 
Esta mensagem foi verificada pelo sistema de antiv�rus e
 acredita-se estar livre de perigo.



[obm-l] Re: [obm-l] Re: [obm-l] Combinatória

2020-03-13 Por tôpico Prof. Douglas Oliveira
Já foi respondia de duas formas aqui.

https://www.mail-archive.com/obm-l@mat.puc-rio.br/msg50069.html

Em sex, 13 de mar de 2020 19:36, Daniel Jelin 
escreveu:

> Uma solução, braçal:
>
> 1) Começamos com 3 ingleses. Há 35 maneiras de colocar outros 6 cidadãos,
> indistintamente, de modo a garantir que 2 deles estejam separando os três
> ingleses: é uma combinação com repetição para escolher, entre 4
> possibilidades, a posição de 4 indivíduos, ou seja, CR4,4 = C7,4 = 35.
> Ilustrando os ingleses por um traço, são essas as maneiras:
>
> 0-1-1-4
> 0-1-2-3
> 0-1-3-2
> 0-1-4-1
> 0-1-5-0
> 0-2-1-3
> 0-2-2-2
> 0-2-3-1
> 0-2-4-0
> 0-3-1-2
> 0-3-2-1
> 0-3-3-0
> 0-4-1-1
> 0-4-2-0
> 0-5-1-0
> 1-1-1-3
> 1-1-2-2
> 1-1-3-1
> 1-1-4-0
> 1-2-1-1
> 1-2-2-1
> 1-2-3-0
> 1-3-1-1
> 1-3-2-0
> 1-4-1-0
> 2-1-1-2
> 2-1-2-1
> 2-1-3-0
> 2-2-1-1
> 2-2-2-0
> 2-3-1-0
> 3-1-1-1
> 3-1-2-0
> 4-1-1-0
>
> 2) Para cada uma das 35 maneiras acima, há um certo número de maneiras de
> posicionar franceses (sem fazer distinção entre os franceses) e turcos (sem
> fazer distinção entre os turcos):
>
> Para o caso '0-1-1-4', por exemplo, temos o seguinte: 2 possibilidades
> para colocar 4 cidadãos no fim da fila, de modo a manter separados
> franceses e turcos ('francês-turco-francês-turco' ou
> 'turco-francês-turco-francês'); e 2 possibilidades para escolher a posição
> do terceiro cidadão turco e do terceiro cidadão francês; ou seja, 2 x 2 = 4
> possibilidades; Evidentemente, também são 4 as possibilidades para os casos
> '1-4-1-0', '4-1-1-0', '0-1-4-1', '0-4-1-1', '1-1-4-0'. Total: 6 x 4 = 24
> possibilidades.
>
> Para '0-1-2-3', temos o seguinte: 2 possibilidades para colocar 3 cidadãos
> no fim da fila ('francês-turco-francês' ou 'turco-francês-turco'); 2
> possibilidades para colocar 2 cidadãos juntos ('francês-turco' ou
> 'turco-francês') e uma possibilidade para colocar o turco ou francês que
> sobrou. ou seja, 4 possibilidades. Como são doze os casos análogos
> ('1-3-2-0', '0-3-2-1', '2-1-3-0' etc.), temos 48 possibilidades.
>
> E assim por diante:
>
> Para '0-1-5-0', são 2 possibilidades e dois casos análogos ('0-1-5-0' e
> '0-5-1-0'), então são 4 possibilidades.
>
> Para '0-3-3-0', são 2 possibilidades - e o caso é único.
>
> Para '0-2-2-2', são 8 possibilidades; há duas variações ('0-2-2-2' e
> '2-2-2-0'), total: 16 possibilidades
>
> Para '0-4-2-0', são 4 possibilidades; há duas variações ('0-4-2-0' e
> '0-2-4-0'), total: 8 possiblidades
>
> Para '1-1-1-3', são 6 possibilidades, há quatro variações ('1-1-1-3',
> '1-1-3-1', '1-3-1-1' e '3-1-1-1'), total: 24 possibilidades
>
> Para '1-1-2-2', são 8 possibilidades, há 6 variações ('1-1-2-2',
> '1-2-2-1', '2-1-1-2', '2-2-1-1', '2-1-2-1' e '1-2-1-2'), total de 48
> possibilidades.
>
> 3) Somando todas as possibilidades, temos 24+48+4+2+16+8+24+48=174
> possibilidades.
>
> 4) Agora vamos permutar os três ingleses (6 possibilidades), os três
> turcos (6 possibilidades) e os três franceses (6 possibilidades). Total:
> 216 possibilidades
>
> 5) Então temos 174 * 216 = 37584 possibilidades
>
> On Fri, Mar 13, 2020 at 9:22 AM Vanderlei Nemitz 
> wrote:
>
>> Bom dia!
>> Não sei se minha mensagem chegou para vocês.
>> Por via das dúvidas, te encaminho.
>>
>> Alguém tem uma ideia para esse problema?
>>
>> Muito obrigado!
>>
>> De quantos modos se podem sentar em fila, 3 ingleses, 3 franceses e 3
>> turcos, de modo que não fiquem dois compatriotas juntos?
>>
>>
>> A resposta é 37584.
>>
>>
>> --
>> Esta mensagem foi verificada pelo sistema de antivírus e
>> acredita-se estar livre de perigo.
>
>
> --
> Esta mensagem foi verificada pelo sistema de antivírus e
> acredita-se estar livre de perigo.

-- 
Esta mensagem foi verificada pelo sistema de antiv�rus e
 acredita-se estar livre de perigo.



[obm-l] Re: [obm-l] Re: [obm-l] Ajuda com dízima

2020-03-08 Por tôpico Prof. Douglas Oliveira
3^2003 é o período certo??, o número de dígitos disso que seria a pergunta.


Douglas oliveira

Em dom, 8 de mar de 2020 11:13, Prof. Douglas Oliveira <
profdouglaso.del...@gmail.com> escreveu:

> Olá Pedro, primeiramente muito obrigado pela sua solução, eu dei uma
> olhada rápida e acredito estar correta. Estarei olhando com mais calma,
> assim que tiver um tempinho.
>
> Douglas Oliveira.
>
> Em dom, 8 de mar de 2020 11:05, Pedro José  escreveu:
>
>> Bom dia!
>> Não compreendi o porquê dessa questão ter sido vilipendiada. Não sou
>> matemático, sou pitaqueiro, ouço falar em inteiros de Gauss vou atrás, de
>> espaço fibrado idem, equações de Pell idem..., o que não consigo aprender
>> fica para o futuro. Quando me aposentar  cursar uma faculdade de
>> matemática. Portanto, nem tudo que resolvo me dá segurança. Reforço, alguém
>> poderia me informar se está correto?
>> Saudações,
>> PJMS.
>>
>> Em ter, 3 de mar de 2020 12:03, Pedro José 
>> escreveu:
>>
>>> Boa tarde!
>>> Não me senti muito seguro na resposta. Está correto?
>>>
>>> Saudações,
>>> PJMS
>>>
>>> Em seg., 2 de mar. de 2020 às 23:27, Pedro José 
>>> escreveu:
>>>
>>>> Boa noite!
>>>> Creio ter conseguido.
>>>> Seja k o número de algarismos do período de 1/3^2005. Como (3,10)=1
>>>> então k é a ordem 10 mod 3^2005.
>>>> 3^(n-2)|| 3^(n-2); (|| significa divide exatamente) e 3^2||10-1 então
>>>> pelo lema de Hensel 3^n||10^(3^(n-2))-1 para n>=2.(i)
>>>> Então 10^(3^(n-2))= 1 mod 3^n logo ord 10 mod 3^n | 3^(n-2) Se
>>>> x<>3^(n-2) absurdo; pois, teria que ser 3^k com k>>> e por (i) 3^(k+2)||10^(3^k)-1 e k+2>>> ord 10 mod 3^2005 =3^2003
>>>> 3^2003 algarismos
>>>> Saudações,
>>>> PJMS
>>>>
>>>> Em sáb, 29 de fev de 2020 16:13, Pedro José 
>>>> escreveu:
>>>>
>>>>> Boa tarde!
>>>>> 3^2005 e não 10^2005.
>>>>>
>>>>> Em sex, 28 de fev de 2020 16:06, Pedro José 
>>>>> escreveu:
>>>>>
>>>>>> Boa tarde!
>>>>>> Questão complicada.
>>>>>> Como (3^2005; 10) =1, o número de dígitos x deve ser a ordem de 10
>>>>>> mod 10^2005. Portanto x | 2*3^2004.
>>>>>> Se 10 fosse uma raiz primitiva de 3^2005 aí daria x=2.3^2004. Mas
>>>>>> parece que não...
>>>>>> Achar essa ordem é muito difícil, pelo menos para mim.
>>>>>> O que achei empiricamente foi a conjectura: ord 10 mod 3^n = 3^(n-2)
>>>>>> para n>=2.
>>>>>> Será que sai por indução, aí seriam 3^2003 algarismos. Caso a
>>>>>> conjectura esteja correta.
>>>>>>
>>>>>> Saudações,
>>>>>> PJMS
>>>>>>
>>>>>> Em qui., 20 de fev. de 2020 às 18:12, Prof. Douglas Oliveira <
>>>>>> profdouglaso.del...@gmail.com> escreveu:
>>>>>>
>>>>>>> Qual o número de dígitos do período de 1/(3^2005) ?
>>>>>>>
>>>>>>>
>>>>>>> Saudações
>>>>>>> Douglas Oliveira
>>>>>>>
>>>>>>> --
>>>>>>> Esta mensagem foi verificada pelo sistema de antivírus e
>>>>>>> acredita-se estar livre de perigo.
>>>>>>
>>>>>>
>> --
>> Esta mensagem foi verificada pelo sistema de antivírus e
>> acredita-se estar livre de perigo.
>
>

-- 
Esta mensagem foi verificada pelo sistema de antiv�rus e
 acredita-se estar livre de perigo.



[obm-l] Re: [obm-l] Re: [obm-l] Ajuda com dízima

2020-03-08 Por tôpico Prof. Douglas Oliveira
Olá Pedro, primeiramente muito obrigado pela sua solução, eu dei uma olhada
rápida e acredito estar correta. Estarei olhando com mais calma, assim que
tiver um tempinho.

Douglas Oliveira.

Em dom, 8 de mar de 2020 11:05, Pedro José  escreveu:

> Bom dia!
> Não compreendi o porquê dessa questão ter sido vilipendiada. Não sou
> matemático, sou pitaqueiro, ouço falar em inteiros de Gauss vou atrás, de
> espaço fibrado idem, equações de Pell idem..., o que não consigo aprender
> fica para o futuro. Quando me aposentar  cursar uma faculdade de
> matemática. Portanto, nem tudo que resolvo me dá segurança. Reforço, alguém
> poderia me informar se está correto?
> Saudações,
> PJMS.
>
> Em ter, 3 de mar de 2020 12:03, Pedro José  escreveu:
>
>> Boa tarde!
>> Não me senti muito seguro na resposta. Está correto?
>>
>> Saudações,
>> PJMS
>>
>> Em seg., 2 de mar. de 2020 às 23:27, Pedro José 
>> escreveu:
>>
>>> Boa noite!
>>> Creio ter conseguido.
>>> Seja k o número de algarismos do período de 1/3^2005. Como (3,10)=1
>>> então k é a ordem 10 mod 3^2005.
>>> 3^(n-2)|| 3^(n-2); (|| significa divide exatamente) e 3^2||10-1 então
>>> pelo lema de Hensel 3^n||10^(3^(n-2))-1 para n>=2.(i)
>>> Então 10^(3^(n-2))= 1 mod 3^n logo ord 10 mod 3^n | 3^(n-2) Se
>>> x<>3^(n-2) absurdo; pois, teria que ser 3^k com k>> e por (i) 3^(k+2)||10^(3^k)-1 e k+2>> ord 10 mod 3^2005 =3^2003
>>> 3^2003 algarismos
>>> Saudações,
>>> PJMS
>>>
>>> Em sáb, 29 de fev de 2020 16:13, Pedro José 
>>> escreveu:
>>>
>>>> Boa tarde!
>>>> 3^2005 e não 10^2005.
>>>>
>>>> Em sex, 28 de fev de 2020 16:06, Pedro José 
>>>> escreveu:
>>>>
>>>>> Boa tarde!
>>>>> Questão complicada.
>>>>> Como (3^2005; 10) =1, o número de dígitos x deve ser a ordem de 10 mod
>>>>> 10^2005. Portanto x | 2*3^2004.
>>>>> Se 10 fosse uma raiz primitiva de 3^2005 aí daria x=2.3^2004. Mas
>>>>> parece que não...
>>>>> Achar essa ordem é muito difícil, pelo menos para mim.
>>>>> O que achei empiricamente foi a conjectura: ord 10 mod 3^n = 3^(n-2)
>>>>> para n>=2.
>>>>> Será que sai por indução, aí seriam 3^2003 algarismos. Caso a
>>>>> conjectura esteja correta.
>>>>>
>>>>> Saudações,
>>>>> PJMS
>>>>>
>>>>> Em qui., 20 de fev. de 2020 às 18:12, Prof. Douglas Oliveira <
>>>>> profdouglaso.del...@gmail.com> escreveu:
>>>>>
>>>>>> Qual o número de dígitos do período de 1/(3^2005) ?
>>>>>>
>>>>>>
>>>>>> Saudações
>>>>>> Douglas Oliveira
>>>>>>
>>>>>> --
>>>>>> Esta mensagem foi verificada pelo sistema de antivírus e
>>>>>> acredita-se estar livre de perigo.
>>>>>
>>>>>
> --
> Esta mensagem foi verificada pelo sistema de antivírus e
> acredita-se estar livre de perigo.

-- 
Esta mensagem foi verificada pelo sistema de antiv�rus e
 acredita-se estar livre de perigo.



Re: [obm-l] obm U

2020-02-22 Por tôpico Prof. Douglas Oliveira
Teoria dos números, combinatória, Geometria, análise, cálculo e álgebra.



Em sáb, 22 de fev de 2020 13:07, Israel Meireles Chrisostomo <
israelmchrisost...@gmail.com> escreveu:

>
> Acho q eu não me fiz entender. Então eu quero saber só a matéria que cai
> na obm nível U, tipo análise, álgebra, topologia, teoria dos números, etc
>
> O
>
> --
> Israel Meireles Chrisostomo
>
> --
> Esta mensagem foi verificada pelo sistema de antivírus e
> acredita-se estar livre de perigo.

-- 
Esta mensagem foi verificada pelo sistema de antiv�rus e
 acredita-se estar livre de perigo.



[obm-l] Ajuda com dízima

2020-02-20 Por tôpico Prof. Douglas Oliveira
Qual o número de dígitos do período de 1/(3^2005) ?


Saudações
Douglas Oliveira

-- 
Esta mensagem foi verificada pelo sistema de antiv�rus e
 acredita-se estar livre de perigo.



[obm-l] Ajuda em duas questões (Geometria plana e equação funcional)

2019-12-13 Por tôpico Prof. Douglas Oliveira
1) Dado um triângulo equilátero ABC, e os segmentos internos de reta BS, CT
e AR tais que BS=CT=AR e além disso B, R, S estão alinhados, C, S, T estão
alinhados e A, T, R estão alinhados, mostre que o triângulo RST também é
equilátero.

2) Essa é a questão da (IMO shortlisted 2008)
  . Find all functions f : (0, ∞) → (0, ∞) such that [(f(p))^2 + (f(q))^2]/
f(r^2 ) + f(s^2 ) = (p^2 + q^2)/(r^2 + s^2) for all p, q, r, s > 0 with pq
= rs.

Pois bem, a minha dúvida é , eu cheguei em duas soluções f(x)=x e f(x)=1/x,
e a minha pergunta seria , precisa mostrar que são as únicas soluções?

Saudações
Douglas Oliveira.

-- 
Esta mensagem foi verificada pelo sistema de antiv�rus e
 acredita-se estar livre de perigo.



[obm-l] Teoria dos números

2019-12-13 Por tôpico Prof. Douglas Oliveira
Olá caros amigos,
preciso de uma ajuda pra criar uma fórmula que seja congruente (módulo p)
ao somatório
S_a=sum{(a^k)/k}, com k de 1 a p-1, sendo p primo ímpar.

Saudações
Douglas Oliveira

-- 
Esta mensagem foi verificada pelo sistema de antiv�rus e
 acredita-se estar livre de perigo.



[obm-l] Números complexos (valor mínimo)

2019-12-13 Por tôpico Prof. Douglas Oliveira
Olá amigos, gostaria de uma ajuda.
Sem usar derivadas...
Como calcular o valor mínimo de lz^4+z+1/2l^2 onde o modelo de z vale 1.

Saudacoes
Douglas Oliveira

-- 
Esta mensagem foi verificada pelo sistema de antiv�rus e
 acredita-se estar livre de perigo.



[obm-l] Funcional equation

2019-12-09 Por tôpico Prof. Douglas Oliveira
Olá, como podemos achar todos os polinômios que satisfazem

P(x^2+1)=[P(x)]^2+1


Saudacoes
Douglas Oliveira

-- 
Esta mensagem foi verificada pelo sistema de antiv�rus e
 acredita-se estar livre de perigo.



Re: [obm-l]

2019-11-30 Por tôpico Prof. Douglas Oliveira
Então, parece que existe sim, de uma olhada aqui
http://mathworld.wolfram.com/EquilateralTriangle.html

Gardner 1977 e guy 1994, além da fórmula existem soluções inteiras para tal
equação.

Abraço
Douglas Oliveira

Em sex., 29 de nov. de 2019 às 20:12, Esdras Muniz <
esdrasmunizm...@gmail.com> escreveu:

> Tentei fazer o mesmo com R=1e l=√3, mas desisti qdo vi o tamanho das
> contas.
>
> Em sex, 29 de nov de 2019 16:09, Claudio Buffara <
> claudio.buff...@gmail.com> escreveu:
>
>> Acho que com números complexos e alguma álgebra sai.
>>
>> Se os vértices do triângulo forem R, Rw  e Rw^2 (onde w = cis(2pi/3) e R
>> é um real positivo) e P = z, então:
>> a = |z - R|, b = |z - Rw|; c = |z - Rw^2| ==>
>> a^2 + b^2 + c^2 = |z - R|^2 + |z - Rw|^2 + |z - Rw^2|^2 = 3*|z|^2 +
>> 3*R^2   (se não errei nenhuma conta)
>>
>> Neste caso, L^2 = 3*R^2, de modo que o lado direito da expressão do
>> enunciado será igual a (3*|z|^2 + 6*R^2)^2 = 9*(|z|^4 + 4*R^2*|z|^2 +
>> 4*R^4).
>>
>> O lado esquerdo deve dar um pouco mais de trabalho...
>>
>>
>> On Tue, Nov 26, 2019 at 7:00 PM gilberto azevedo 
>> wrote:
>>
>>> Pesquisando achei uma relação muito interessante, mas não achei nenhuma
>>> demonstração dela na web.
>>> Pra quem se interessar Seja um ponto P no interior de um triângulo
>>> equilátero de lado l, e a,b,c a distância desse ponto aos vértices do
>>> triângulo. Provar que :
>>> 3( a⁴ + b⁴ + c⁴ + l⁴) = ( a² + b² + c² + l²)²
>>>
>>> --
>>> Esta mensagem foi verificada pelo sistema de antivírus e
>>> acredita-se estar livre de perigo.
>>
>>
>> --
>> Esta mensagem foi verificada pelo sistema de antivírus e
>> acredita-se estar livre de perigo.
>
>
> --
> Esta mensagem foi verificada pelo sistema de antivírus e
> acredita-se estar livre de perigo.

-- 
Esta mensagem foi verificada pelo sistema de antiv�rus e
 acredita-se estar livre de perigo.



Re: [obm-l]

2019-11-12 Por tôpico Prof. Douglas Oliveira
Será que não sai usando somente congruência módulo 8?

Em ter., 12 de nov. de 2019 às 20:07, Pedro José 
escreveu:

> Boa noite!
> Esdras,
> tem como você postar, mesmo para o caso apenas de n par?
>
> Grato!
>
> Saudações,
> PJMS.
>
> Em ter., 12 de nov. de 2019 às 19:52, Pedro José 
> escreveu:
>
>> Boa noite!
>> Carlos Gustavo,
>> grato pela luz, estava tão obsecado e só rodando em círculos, tal qual
>> patrulha perdida.
>>
>> Saudações,
>> PJMS
>>
>> Em ter., 12 de nov. de 2019 às 19:19, Esdras Muniz <
>> esdrasmunizm...@gmail.com> escreveu:
>>
>>> Dá para mostrar que a única solução com a e b pares é (2, 2). Agora com
>>> a e b ímpares, não consegui.
>>>
>>> Em ter, 12 de nov de 2019 18:19, Pedro José 
>>> escreveu:
>>>
>>>> Boa noite!
>>>> Agora captei vosso pensamento.
>>>> Só que ao transformar a equação em uma equação de Pell, nós maculamos a
>>>> função 3^n.
>>>> Em verdade a solução para a par a= 2n, seria (2,2); pois, como
>>>> mencionara anteriormente se a é par, b também o é.
>>>> Só que quando procuramos as outras soluções, baseando-se na propriedade
>>>> de que a norma em Q [RAiz(A)] conserva a multiplicação. Só que quando eu
>>>> pego a solução
>>>> 3 + 2 Raiz(2) e elevo ao quadrado 17 + 12 Raiz(2). Se eu pegar
>>>> 17^2-2*12^2=1 eu atendo x^2 - 2Y^2=1. E assim sucessivamente. Mas não
>>>> existe n inteiro tal que 3^n=17, então não é uma solução da equação
>>>> original.
>>>> Creio que seja um pouco mais complicada a solução. Pois o difícil é
>>>> saber quando atende também a 3^n.
>>>> Acredito que deva haver uma forma de restringir a essas soluções, pois,
>>>> definir em que condições a solução terá x como uma potência de 3 seja bem
>>>> difícil.
>>>> Estou apanhando mais do que mala velha em véspera de viagem.
>>>> Se alguém postar uma solução, me ajudaria bastante.
>>>>
>>>> Saudações,
>>>> PJMS
>>>>
>>>>
>>>> Saudações,
>>>> PJMS.
>>>>
>>>>
>>>>
>>>> Em ter., 12 de nov. de 2019 às 17:25, Pedro José 
>>>> escreveu:
>>>>
>>>>> Boa tarde!
>>>>> Douglas,
>>>>> perdoe-me pela minha miopia, mas você poderia detalhar melhor onde
>>>>> entra a equação de Pell?
>>>>> A equação de Pell não é x^2-Dy^2 = N?
>>>>> Se a é par b é par e se a ímpar b é ímpar para atender mod8,
>>>>> Não consegui captar a sugestão.
>>>>>
>>>>> Saudações,
>>>>> PJMS
>>>>>
>>>>> Em ter., 12 de nov. de 2019 às 16:50, Prof. Douglas Oliveira <
>>>>> profdouglaso.del...@gmail.com> escreveu:
>>>>>
>>>>>> Hum, então, vamos analisar o caso de a ser par do tipo 2n.
>>>>>>
>>>>>> Assim podemos escrever que (3^n+b(sqrt2))(3^n-b(sqrt2))=1
>>>>>> Dai através da solução mínima que o Pedro fez, como (1,1) por
>>>>>> exemplo, da pra ver que são infinitas soluções usando a equação de Pell.
>>>>>>
>>>>>> Abraco
>>>>>> Douglas Oliveira.
>>>>>>
>>>>>>
>>>>>>
>>>>>> Em dom, 10 de nov de 2019 19:33, gilberto azevedo <
>>>>>> gil159...@gmail.com> escreveu:
>>>>>>
>>>>>>> [HELP]
>>>>>>>
>>>>>>> Achas todos os pares (a,b) inteiros positivos tais que :
>>>>>>> 3^a = 2b² + 1.
>>>>>>>
>>>>>>>
>>>>>>> --
>>>>>>> Esta mensagem foi verificada pelo sistema de antivírus e
>>>>>>> acredita-se estar livre de perigo.
>>>>>>
>>>>>>
>>>>>> --
>>>>>> Esta mensagem foi verificada pelo sistema de antivírus e
>>>>>> acredita-se estar livre de perigo.
>>>>>
>>>>>
>>>> --
>>>> Esta mensagem foi verificada pelo sistema de antivírus e
>>>> acredita-se estar livre de perigo.
>>>
>>>
>>> --
>>> Esta mensagem foi verificada pelo sistema de antivírus e
>>> acredita-se estar livre de perigo.
>>
>>
> --
> Esta mensagem foi verificada pelo sistema de antivírus e
> acredita-se estar livre de perigo.

-- 
Esta mensagem foi verificada pelo sistema de antiv�rus e
 acredita-se estar livre de perigo.



Re: [obm-l]

2019-11-12 Por tôpico Prof. Douglas Oliveira
*Vamos deixar a preguiça  um pouco de lado, decidi escrever um pouco.*

*Equações de Pell são equações diofantinas não lineares da forma  x2 – Dy2
= m, onde D é um número natural e m um número inteiro. Se m = 1 temos a
equação  x2 – Dy2 = 1, onde notamos que estas equações possuem 2 soluções
inteiras triviais, x = 1, y = 0 e x = – 1 e y = 0. Fora estas soluções,
todas as outras soluções inteiras podem ser arranjadas em conjuntos de 4
soluções, onde apenas permutamos os sinais dos números. Por exemplo, desde
que (3, 2) é uma solução da equação  x2 – 2y2 = 1, também temos as soluções
inteiras (– 3, 2), (– 3, – 2) e (3, – 2). Evidentemente, em toda classe de
soluções existe uma onde x e y são naturais. Denominemos estas solução de
soluções naturais da Equação de Pell. Claramente, para determinar as
soluções de uma Equação de Pell basta determinar as soluções naturais.*

*O caso em que m = 1 e D for um quadrado perfeito (D = n2) não é
interessante, pois assim a equação pode ser reescrita da forma:  x2 – n2y2
= (x – ny)(x + ny) = 1  onde equação não possui soluções naturais fora à
trivial  x = 1 e y = 0.*


*Por exemplo, pode-se observar que a Equação de Pell  x2 – 3y2 = 1  possui
uma menor solução natural  x0 = 2, e y0 = 1. Deste modo, pode-se encontrar
uma outra solução natural, fazendo  x1 = x02 + 3y02 = 7  e  y1 = 2x0y0 = 4.
Conferindo, temos evidentemente que  72 – 2.42 = 1. Agora temos (1, 0), (2,
1) e (7, 4) como soluções naturais de  x2 – 3y2 = 1. Para encontrar outra
basta fazer  x2 = x12 + 3y12 = 97  e  y2 = 2x1y1 = 28. Conferindo, notamos
que realmente  972 – 3.282 = 1. E assim por diante, onde podemos fazer este
procedimento de cálculos infinitas vezes, obtendo infinitas soluções
naturais para a Equação de Pell  x2 – 3y2 = 1.*

*Todas as soluções de x2 – 3y2 = 1 podem ser encontradas através da
expressão x_n+y_n(sqrt(3))=(x_o+y_o(sqrt(3))^n , ou seja,
 x_n+y_n(sqrt(3))=(2+sqrt(3))^n.*


Grande abraço

Douglas Oliveira



Em dom., 10 de nov. de 2019 às 19:33, gilberto azevedo 
escreveu:

> [HELP]
>
> Achas todos os pares (a,b) inteiros positivos tais que :
> 3^a = 2b² + 1.
>
>
> --
> Esta mensagem foi verificada pelo sistema de antivírus e
> acredita-se estar livre de perigo.

-- 
Esta mensagem foi verificada pelo sistema de antiv�rus e
 acredita-se estar livre de perigo.



Re: [obm-l]

2019-11-12 Por tôpico Prof. Douglas Oliveira
Hum, então, vamos analisar o caso de a ser par do tipo 2n.

Assim podemos escrever que (3^n+b(sqrt2))(3^n-b(sqrt2))=1
Dai através da solução mínima que o Pedro fez, como (1,1) por exemplo, da
pra ver que são infinitas soluções usando a equação de Pell.

Abraco
Douglas Oliveira.



Em dom, 10 de nov de 2019 19:33, gilberto azevedo 
escreveu:

> [HELP]
>
> Achas todos os pares (a,b) inteiros positivos tais que :
> 3^a = 2b² + 1.
>
>
> --
> Esta mensagem foi verificada pelo sistema de antivírus e
> acredita-se estar livre de perigo.

-- 
Esta mensagem foi verificada pelo sistema de antiv�rus e
 acredita-se estar livre de perigo.



[obm-l] Re: [obm-l] Re: [obm-l] Séries e somatórios

2019-10-31 Por tôpico Prof. Douglas Oliveira
Gosto muito do manual de sequências e séries do Luis Lopes.

Douglas Oliveira.

Em qua, 30 de out de 2019 20:19, Esdras Muniz 
escreveu:

> O livro concrete mathematics fala disso.
>
> Em qua, 30 de out de 2019 19:51, Alexandre Antunes <
> prof.alexandreantu...@gmail.com> escreveu:
>
>>
>> Boa noite,
>>
>> Alguém tem alguma referência de livro/apostila sobre operações e
>> propriedades "avançadas" sobre séries, somatórios, somatórios duplos, etc...
>>
>> Antecipadamente agradeço.
>>
>> Atenciosamente,
>>
>> Prof. Msc. Alexandre Antunes
>> www alexandre antunes com br
>>
>> --
>> Esta mensagem foi verificada pelo sistema de antivírus e
>> acredita-se estar livre de perigo.
>
>
> --
> Esta mensagem foi verificada pelo sistema de antivírus e
> acredita-se estar livre de perigo.

-- 
Esta mensagem foi verificada pelo sistema de antiv�rus e
 acredita-se estar livre de perigo.



Re: [obm-l] Re: Problema 19 da OMDF de 2018.

2019-10-25 Por tôpico Prof. Douglas Oliveira
Vamos fazer por complexos.

1) Coloque os eixos real e imaginário com origem no vértice A.

2) Chame de z1 o complexo AP  e de z2 o complexo AQ.

3)Faca uma rotação de 60 graus, z1cis(60)=z2.

4) Igualando as partes real e imaginaria teremos para resposta 2b-a3^(1/2)

Abraço
ProfDouglasOliveira

Em qui, 24 de out de 2019 23:44, Guilherme Abbehusen <
gui.abbehuse...@gmail.com> escreveu:

> Aliás, esqueci de avisar que a resposta deve ser em função de a e b. As
> alternativas seriam: A) 2a - b*3^1/2B) a - 2b*3^1/2 C) 3b -
> a*3^1/2D) 2b - a*3^1/2 E) b - a*3^1/2
>
>
> Em qui, 24 de out de 2019 às 23:06, Guilherme Abbehusen <
> gui.abbehuse...@gmail.com> escreveu:
>
>> Olá, alguém poderia me ajudar com essa questão?
>>
>> Azambuja tem uma folha retangular ABCD de dimensões AB = a e BC = b , na
>> qual quer efetuar três cortes para obter um triângulo equilátero. Portanto,
>> escolhe o ponto P sobre BC e o ponto Q sobre CD, obtendo o triângulo
>> equilátero APQ. Qual é o comprimento do segmento BP?
>>
>> Agradeço desde já.
>>
>
> --
> Esta mensagem foi verificada pelo sistema de antivírus e
> acredita-se estar livre de perigo.

-- 
Esta mensagem foi verificada pelo sistema de antiv�rus e
 acredita-se estar livre de perigo.



[obm-l] Re: [obm-l] Re: [obm-l] Equação exponencial

2019-10-16 Por tôpico Prof. Douglas Oliveira
Pocha, explicadissimo, thank you my friend.

Em qua, 16 de out de 2019 18:12, Ralph Teixeira 
escreveu:

> Depende!
>
> (Esta discussao eh analoga aaquela outra de "Afinal, 0 eh natural ou
> nao?"... cuja resposta eh "Decida como quiser, diga para todos como voce
> decidiu, e seja coerente. De preferencia, escreva as coisas para evitar a
> pergunta.")
>
> O problema eh a convenção: quanto vale 0^0 ? Ha duas opções: alguns
> matemáticos usam que 0^0=1; outros (acho que a maioria?) preferem dizer que
> 0^0 nao eh uma operação permitida.
>
> Eu pessoalmente prefiro dizer que 0^0=1 (sou minoria?). Veja bem, eh uma
> convenção, apenas uma convenção, então não tem "certo" ou "errado"... Mas
> tenho alguns argumentos a favor disto:
> A1) Se f(x) e g(x) sao funcoes **analiticas** em torno de x=a, com lim
> f(x) = lim g(x) = 0 quando x->a, e f nao eh identicamente nula perto de a,
> entao lim f^g=1 quando x->a. Por este motivo, 99% dos exercicios de Calculo
> que caem numa "indeterminacao" do tipo 0^0 acabam dando 1!
> A2) Com esta convencao, a funcao f(x)=x^0 vale 1 para todo x real, sem
> excecao.
> A3) Tecnicamente, (A2) de novo, mas agora explicando onde isso eh util:
> para descrever um polinomio generico (ou uma serie de potencias, que a
> gente usa bastante para resolver algumas EDOs), a gente escreve SUM (k=0 a
> n) a_k x^k (ou SUM (k=0 a Inf) a_k x^k) -- aqui SUM eh um somatório. Pois
> bem, o primeiro termo ali, quando k=0, eh a_0.x^0, e eu quero que isso
> valha a_0 para todo x, inclusive para x=0. Se voce eh da escola do "0^0 nao
> eh permitido", você vai ter que escrever o a_0 fora do somatório sozinho,
> ou abrir uma exceção, ou fingir que nao viu o problema. :(
>
> Para fazer o contraponto, vejo argumentos a favor de definir 0^0 como
> "operacao invalida":
> B1) A funcao g(x,y)=x^y (x>0, y>0) NAO TEM LIMITE quando (x,y)->(0,0),
> então nao faz sentido botar um valor especifico para g(0,0).
> B2) Ok, 99% dos limites do tipo 0^0 dao 1, mas os outros 1% NAO DAO 1, e
> isto poderia causar confusao!
> B3) A funcao f(x)=0^x eh continua em (0,Inf). Colocando f(0)=1, ela fica
> descontinua em x=0.
>
> Ainda assim, prefiro 0^0=1 -- acho (A3) forte, acho MUITO mais conveniente
> pensar que 0^0=1 para nao ter que me separar aquele a_0 do polinomio.
>
> Abraco, Ralph.
>
> On Wed, Oct 16, 2019 at 4:36 PM Prof. Douglas Oliveira <
> profdouglaso.del...@gmail.com> wrote:
>
>> Amigos, me ajudem por favor.
>>
>> Afinal de contas, zero, é ou não é raiz da equação
>> (sqrt(x))^x=x^(sqrt(x=)?
>>
>> Douglas Oliveira.
>>
>> --
>> Esta mensagem foi verificada pelo sistema de antivírus e
>> acredita-se estar livre de perigo.
>
>
> --
> Esta mensagem foi verificada pelo sistema de antivírus e
> acredita-se estar livre de perigo.

-- 
Esta mensagem foi verificada pelo sistema de antiv�rus e
 acredita-se estar livre de perigo.



[obm-l] Equação exponencial

2019-10-16 Por tôpico Prof. Douglas Oliveira
Amigos, me ajudem por favor.

Afinal de contas, zero, é ou não é raiz da equação (sqrt(x))^x=x^(sqrt(x=)?

Douglas Oliveira.

-- 
Esta mensagem foi verificada pelo sistema de antiv�rus e
 acredita-se estar livre de perigo.



Re: [obm-l] Trigonometria

2019-08-28 Por tôpico Prof. Douglas Oliveira
Opa mandei errado aqui a tangente, não é dessa questão não, essa questão
sua tem algo errado.樂樂

Em qua, 28 de ago de 2019 14:42, Carlos Monteiro <
cacacarlosalberto1...@gmail.com> escreveu:

> Pode enviar a solução?
>
> Em qua, 28 de ago de 2019 13:57, Prof. Douglas Oliveira <
> profdouglaso.del...@gmail.com> escreveu:
>
>> X=arctg(2/3raiz5)
>>
>> Em qua, 28 de ago de 2019 10:13, Carlos Monteiro <
>> cacacarlosalberto1...@gmail.com> escreveu:
>>
>>> Sim, EC=2x; DE=x; BD=x.
>>>
>>> Em qua, 28 de ago de 2019 08:56, Claudio Buffara <
>>> claudio.buff...@gmail.com> escreveu:
>>>
>>>> Ou seja, os pontos ocorrem na ordem B-D-E-C, E é o ponto médio de BC e
>>>> D é o ponto médio de BE. É isso?
>>>>
>>>> On Wed, Aug 28, 2019 at 8:15 AM Carlos Monteiro <
>>>> cacacarlosalberto1...@gmail.com> wrote:
>>>>
>>>>> Caramba, me desculpa
>>>>>
>>>>> O correto é 2(BD)=2(DE)=EC
>>>>>
>>>>> Em ter, 27 de ago de 2019 11:24, Prof. Douglas Oliveira <
>>>>> profdouglaso.del...@gmail.com> escreveu:
>>>>>
>>>>>> Tu tem a fonte dela amigao??
>>>>>> A notação é essa mesmo 2(BD)=2(DE)=2(EC)?
>>>>>>
>>>>>> Em ter, 27 de ago de 2019 às 09:48, Carlos Monteiro <
>>>>>> cacacarlosalberto1...@gmail.com> escreveu:
>>>>>>
>>>>>>> Seja ABC um triângulo. Sejam D e E pontos no lado BC tal que
>>>>>>> 2(BD)=2(DE)=2(EC). Sabendo que os círculos inscritos nos triângulos ABD,
>>>>>>> ADE e AEC têm o mesmo raio, calcule o seno do ângulo .
>>>>>>>
>>>>>>>
>>>>>>> --
>>>>>>> Esta mensagem foi verificada pelo sistema de antivírus e
>>>>>>> acredita-se estar livre de perigo.
>>>>>>
>>>>>>
>>>>>> --
>>>>>> Esta mensagem foi verificada pelo sistema de antivírus e
>>>>>> acredita-se estar livre de perigo.
>>>>>
>>>>>
>>>>> --
>>>>> Esta mensagem foi verificada pelo sistema de antivírus e
>>>>> acredita-se estar livre de perigo.
>>>>
>>>>
>>>> --
>>>> Esta mensagem foi verificada pelo sistema de antivírus e
>>>> acredita-se estar livre de perigo.
>>>
>>>
>>> --
>>> Esta mensagem foi verificada pelo sistema de antivírus e
>>> acredita-se estar livre de perigo.
>>
>>
>> --
>> Esta mensagem foi verificada pelo sistema de antivírus e
>> acredita-se estar livre de perigo.
>
>
> --
> Esta mensagem foi verificada pelo sistema de antivírus e
> acredita-se estar livre de perigo.

-- 
Esta mensagem foi verificada pelo sistema de antiv�rus e
 acredita-se estar livre de perigo.



Re: [obm-l] Trigonometria

2019-08-28 Por tôpico Prof. Douglas Oliveira
X=arctg(2/3raiz5)

Em qua, 28 de ago de 2019 10:13, Carlos Monteiro <
cacacarlosalberto1...@gmail.com> escreveu:

> Sim, EC=2x; DE=x; BD=x.
>
> Em qua, 28 de ago de 2019 08:56, Claudio Buffara <
> claudio.buff...@gmail.com> escreveu:
>
>> Ou seja, os pontos ocorrem na ordem B-D-E-C, E é o ponto médio de BC e D
>> é o ponto médio de BE. É isso?
>>
>> On Wed, Aug 28, 2019 at 8:15 AM Carlos Monteiro <
>> cacacarlosalberto1...@gmail.com> wrote:
>>
>>> Caramba, me desculpa
>>>
>>> O correto é 2(BD)=2(DE)=EC
>>>
>>> Em ter, 27 de ago de 2019 11:24, Prof. Douglas Oliveira <
>>> profdouglaso.del...@gmail.com> escreveu:
>>>
>>>> Tu tem a fonte dela amigao??
>>>> A notação é essa mesmo 2(BD)=2(DE)=2(EC)?
>>>>
>>>> Em ter, 27 de ago de 2019 às 09:48, Carlos Monteiro <
>>>> cacacarlosalberto1...@gmail.com> escreveu:
>>>>
>>>>> Seja ABC um triângulo. Sejam D e E pontos no lado BC tal que
>>>>> 2(BD)=2(DE)=2(EC). Sabendo que os círculos inscritos nos triângulos ABD,
>>>>> ADE e AEC têm o mesmo raio, calcule o seno do ângulo .
>>>>>
>>>>>
>>>>> --
>>>>> Esta mensagem foi verificada pelo sistema de antivírus e
>>>>> acredita-se estar livre de perigo.
>>>>
>>>>
>>>> --
>>>> Esta mensagem foi verificada pelo sistema de antivírus e
>>>> acredita-se estar livre de perigo.
>>>
>>>
>>> --
>>> Esta mensagem foi verificada pelo sistema de antivírus e
>>> acredita-se estar livre de perigo.
>>
>>
>> --
>> Esta mensagem foi verificada pelo sistema de antivírus e
>> acredita-se estar livre de perigo.
>
>
> --
> Esta mensagem foi verificada pelo sistema de antivírus e
> acredita-se estar livre de perigo.

-- 
Esta mensagem foi verificada pelo sistema de antiv�rus e
 acredita-se estar livre de perigo.



Re: [obm-l] Trigonometria

2019-08-27 Por tôpico Prof. Douglas Oliveira
Tu tem a fonte dela amigao??
A notação é essa mesmo 2(BD)=2(DE)=2(EC)?

Em ter, 27 de ago de 2019 às 09:48, Carlos Monteiro <
cacacarlosalberto1...@gmail.com> escreveu:

> Seja ABC um triângulo. Sejam D e E pontos no lado BC tal que
> 2(BD)=2(DE)=2(EC). Sabendo que os círculos inscritos nos triângulos ABD,
> ADE e AEC têm o mesmo raio, calcule o seno do ângulo .
>
>
> --
> Esta mensagem foi verificada pelo sistema de antivírus e
> acredita-se estar livre de perigo.

-- 
Esta mensagem foi verificada pelo sistema de antiv�rus e
 acredita-se estar livre de perigo.



[obm-l] Minimizar

2019-08-22 Por tôpico Prof. Douglas Oliveira
Olá amigos ajuda a minimizar a expressão.
sin(x+y)/((1+sinx)(1+siny))

Please

Thank you
Douglas oliveira

-- 
Esta mensagem foi verificada pelo sistema de antiv�rus e
 acredita-se estar livre de perigo.



[obm-l] Re: [obm-l] Somatórios

2019-07-22 Por tôpico Prof. Douglas Oliveira
Manual de sequencias do LUis Lopes, volumes 1 e 2.

Douglas Oliveira

Em sáb, 20 de jul de 2019 às 23:38, Eduardo Henrique 
escreveu:

> Pessoal, podem me indicar algum material que explique como funcionam os
> somatórios? Gostaria de algum que explicasse em que casos podemos inverter
> somatórios, quais as condições... tanto pra finitos quanto pra infinitos.
> Pode ser apenas nomes de livros que tenham isso que eu corro atrás. Valeu!
>
>
> --
> Esta mensagem foi verificada pelo sistema de antivírus e
> acredita-se estar livre de perigo.
>

-- 
Esta mensagem foi verificada pelo sistema de antiv�rus e
 acredita-se estar livre de perigo.



Re: [obm-l] Geometria

2019-07-15 Por tôpico Prof. Douglas Oliveira
Opa , desculpa era quadrado

Em seg, 15 de jul de 2019 22:58, Joao Breno 
escreveu:

> ABCD é um quadrilátero qualquer ou um retângulo?
>
> Att, Breno.
>
> Em seg, 15 de jul de 2019 22:18, Prof. Douglas Oliveira <
> profdouglaso.del...@gmail.com> escreveu:
>
>> Olá amigos podem me ajudar no seguinte problema?
>>
>> Dado um [image: $ABCD$], onde [image: $M,K, L$] e [image: $N$] são
>> pontos nos lados [image: $AB, BC,CD$] e [image: $DA$], respectivamente,
>> tal que [image: $\angle MKA =\angle KAL = \angle ALN = 45^o$]. Prove que 
>> [image:
>> $MK^2 + AL^2 = AK^2 + LN^2$]
>>
>> Att
>> Douglas Oliveira
>>
>> --
>> Esta mensagem foi verificada pelo sistema de antivírus e
>> acredita-se estar livre de perigo.
>
>
> --
> Esta mensagem foi verificada pelo sistema de antivírus e
> acredita-se estar livre de perigo.

-- 
Esta mensagem foi verificada pelo sistema de antiv�rus e
 acredita-se estar livre de perigo.



[obm-l] Geometria

2019-07-15 Por tôpico Prof. Douglas Oliveira
Olá amigos podem me ajudar no seguinte problema?

Dado um [image: $ABCD$], onde [image: $M,K, L$] e [image: $N$] são pontos
nos lados [image: $AB, BC,CD$] e [image: $DA$], respectivamente, tal
que [image:
$\angle MKA =\angle KAL = \angle ALN = 45^o$]. Prove que [image: $MK^2 +
AL^2 = AK^2 + LN^2$]

Att
Douglas Oliveira

-- 
Esta mensagem foi verificada pelo sistema de antiv�rus e
 acredita-se estar livre de perigo.



[obm-l] Re: [obm-l] teoria dos números curiosidade

2019-07-03 Por tôpico Prof. Douglas Oliveira
Lembro-me de uma resolucao feita por amigo aqui da lista, o Carlos Victor,
na eureka número 2, no finalzinho, de uma olhada.

Att
Douglas Oliveira.

Em qua, 3 de jul de 2019 15:08, Israel Meireles Chrisostomo <
israelmchrisost...@gmail.com> escreveu:

> Esses dias eu estava estudando sobre frações unitárias, e assisti a um
> vídeo do pessoal impa sobre o assunto e fiquei sinceramente maravilhado com
> a engenhosidade dos egípcios.Mas uma questão não saiu da minha cabeça: um
> número inteiro pode ser separado em frações unitárias?Quais são as
> propriedades necessárias que uma fração deve ter para ser decomposta em
> frações egípcias
> --
> Israel Meireles Chrisostomo
>
>
> <http://www.avg.com/email-signature?utm_medium=email_source=link_campaign=sig-email_content=webmail>
>  Livre
> de vírus. www.avg.com
> <http://www.avg.com/email-signature?utm_medium=email_source=link_campaign=sig-email_content=webmail>.
> <#m_8002768564935167525_DAB4FAD8-2DD7-40BB-A1B8-4E2AA1F9FDF2>
>
> --
> Esta mensagem foi verificada pelo sistema de antivírus e
> acredita-se estar livre de perigo.

-- 
Esta mensagem foi verificada pelo sistema de antiv�rus e
 acredita-se estar livre de perigo.



[obm-l] Re: [obm-l] Re: [obm-l] Re: [obm-l] Desigualdade com potências

2018-04-29 Por tôpico Douglas Oliveira de Lima
Valeu Ralph, thanks.

Douglas Oliveira.

Em dom, 29 de abr de 2018 16:49, Ralph Teixeira <ralp...@gmail.com>
escreveu:

> Que tal assim:
>
> POR BAIXO (BEM folgado): Como 3^3=27<32=2^5, temos
> 3^100<(3^3)^34<(2^5)^34=2^170. Portanto
> 3^100+2^100<2^170+2^100<2^170+2^170=2^171<2^200=4^100.
> POR CIMA (mais apertado!): Como 3^7=2187>2^11=2048, temos
> 3^100=9.(3^98)>9.(2^154)>(2^3).(2^154)=2^157. Somando 2^100, ficamos abaixo
> de 2.(2^157)=2^158=4^79.
>
> Abraco, Ralph.
>
> 2018-04-29 13:09 GMT-03:00 Anderson Torres <torres.anderson...@gmail.com>:
>
>> 2018-04-29 8:45 GMT-03:00 Douglas Oliveira de Lima
>> <profdouglaso.del...@gmail.com>:
>> > Prove que 4^79<2^100+3^100<4^100, usando matemática elementar.
>> >
>>
>> O desejo de trapacear isso com log é muito forte :)
>>
>> Isso equivale a mostrar que
>>
>> 2^158-2^100<3^100<2^200-2^100
>>
>> Ou
>>
>> (2^58-1)*2^100<3^100<(2^100-1)*2^100
>>
>> Ou talvez
>>
>> 2^58 < (3/2)^100+1 < 2^100
>>
>> Daqui, tenho poucas ideias para a desigualdade mais à esquerda...
>>
>> > Douglas Oliveira.
>> >
>> > --
>> > Esta mensagem foi verificada pelo sistema de antivírus e
>> > acredita-se estar livre de perigo.
>>
>> --
>> Esta mensagem foi verificada pelo sistema de antivírus e
>>  acredita-se estar livre de perigo.
>>
>>
>> =
>> Instruções para entrar na lista, sair da lista e usar a lista em
>> http://www.mat.puc-rio.br/~obmlistas/obm-l.html
>> =
>>
>
>
> --
> Esta mensagem foi verificada pelo sistema de antivírus e
> acredita-se estar livre de perigo.

-- 
Esta mensagem foi verificada pelo sistema de antiv�rus e
 acredita-se estar livre de perigo.



[obm-l] Desigualdade com potências

2018-04-29 Por tôpico Douglas Oliveira de Lima
Prove que 4^79<2^100+3^100<4^100, usando matemática elementar.

Douglas Oliveira.

-- 
Esta mensagem foi verificada pelo sistema de antiv�rus e
 acredita-se estar livre de perigo.



Re: [obm-l] Desigualdade

2018-04-17 Por tôpico Douglas Oliveira de Lima
Nao entendi esse a_k Produto.

por exemplo se fossem a_1, a_2 e a_3, entao seria
1/a_1[(a_3)^2-(a_1)^2][(a_2)^2-(a_1)^2]
+1/a_2[(a_3)^2-(a_2)^2][(a_1)^2-(a_2)^2]+1/a_3[(a_2)^2-(a_3)^2][(a_1)^2-(a_3)^2],
é maior que zero , é isso?

Douglas Oliveira.

Em ter, 17 de abr de 2018 00:49, Artur Costa Steiner <
artur.costa.stei...@gmail.com> escreveu:

> Sejam a_1, a_n  números positivos, distintos dois a dois, e, para k =
> 1, ... n, definamos
>
> p_k = a_k Produto (j = 1, n, j <> k) ((a_j)^2) - (a_k)^2)
>
> Mostre que 1/p_1 ... + 1/p_n > 0
>
> Artur
>
>
>
> --
> Esta mensagem foi verificada pelo sistema de antivírus e
>  acredita-se estar livre de perigo.
>
>
> =
> Instruções para entrar na lista, sair da lista e usar a lista em
> http://www.mat.puc-rio.br/~obmlistas/obm-l.html
> =
>

-- 
Esta mensagem foi verificada pelo sistema de antiv�rus e
 acredita-se estar livre de perigo.



Re: [obm-l] Soma (k = 1, n) 1/P'(r_k) = 0

2018-04-16 Por tôpico Douglas Oliveira de Lima
Entao a questao é até que ponto ela é verdadeira , pois funciona para casos
elementares.

Douglas Oliveira

Em dom, 15 de abr de 2018 22:29, Bernardo Freitas Paulo da Costa <
bernardo...@gmail.com> escreveu:

> 2018-04-15 13:09 GMT-03:00 Douglas Oliveira de Lima
> <profdouglaso.del...@gmail.com>:
> > Usa o polinomio de Lagrange , nao é nada obvia mesmo.
>
> Como usa Lagrange, a fórmula segue para k = 0, 1, ... n-1
> (interpolando em n pontos, vamos até grau n-1).  E é, de fato, falso
> para k = n, use P(x) = (x-1)(x+1).
>
> Além disso, mesmo para k = n-1, a demonstração por complexa não se
> aplica mais (o grau dá errado...), e o mesmo polinômio serve para
> mostrar que a soma não dá mais zero.
>
> Abraços,
> --
> Bernardo Freitas Paulo da Costa
>
> --
> Esta mensagem foi verificada pelo sistema de antivírus e
>  acredita-se estar livre de perigo.
>
>
> =
> Instru�ões para entrar na lista, sair da lista e usar a lista em
> http://www.mat.puc-rio.br/~obmlistas/obm-l.html
> =
>

-- 
Esta mensagem foi verificada pelo sistema de antiv�rus e
 acredita-se estar livre de perigo.



Re: [obm-l] Soma (k = 1, n) 1/P'(r_k) = 0

2018-04-15 Por tôpico Douglas Oliveira de Lima
Usa o polinomio de Lagrange , nao é nada obvia mesmo.

Douglas Oliveira.

Em sex, 13 de abr de 2018 13:41, Claudio Buffara <claudio.buff...@gmail.com>
escreveu:

> Essa identidade:
>  x^k=soma (i=1,...,n)(x_i)^k.P(x)/(x-x_i).P'(x_i)
> não me parece nada óbvia.
>
> []s,
> Claudio.
>
>
> 2018-04-13 5:56 GMT-03:00 Douglas Oliveira de Lima <
> profdouglaso.del...@gmail.com>:
>
>> Entao, sendo x^k=soma (i=1,...,n)(x_i)^k.P(x)/(x-x_i).P'(x_i) , é só
>> igualar os coeficientes de x^(n-1) e pronto, a identidade se torna ate mais
>> genérica
>>
>> Soma (i= 1, n) (x_i)^k/P'(x_i) = 0
>>
>> Obs: x_i sao raizes.
>>
>> Abraco
>>
>> Douglas Oliveira.
>>
>>
>>
>>
>> Em 8 de abr de 2018 20:50, "Artur Steiner" <artur.costa.stei...@gmail.com>
>> escreveu:
>>
>> Seja P um polinômio complexo, de grau n >= 2, que tenha n raízes simples
>> r_1, ... r_n. Mostre que Soma (k = 1, n) 1/P'(r_k) = 0.
>>
>> Para quem conhece um pouco de análise complexa, isto é corolário de um
>> resultado geral. Mas parece que pode ser provado sem análise complexa.
>>
>> Artur Costa Steiner
>>
>> --
>> Esta mensagem foi verificada pelo sistema de antivírus e
>> acredita-se estar livre de perigo.
>>
>>
>>
>> --
>> Esta mensagem foi verificada pelo sistema de antivírus e
>> acredita-se estar livre de perigo.
>>
>
>
> --
> Esta mensagem foi verificada pelo sistema de antivírus e
> acredita-se estar livre de perigo.

-- 
Esta mensagem foi verificada pelo sistema de antiv�rus e
 acredita-se estar livre de perigo.



[obm-l] Re: [obm-l] Re: [obm-l] Combinatória

2018-04-14 Por tôpico Douglas Oliveira de Lima
Entao , veio de quantas soluções inteiras positivas existem para x+yz+w=100.

Douglas Oliveira.

Em sáb, 14 de abr de 2018 13:37, Claudio Buffara <claudio.buff...@gmail.com>
escreveu:

> Que eu saiba, só no braço, mesmo...
>
> n(k) é uma fórmula envolvendo os expoentes da decomposição de k em fatores
> primos.
> Não conheço nenhuma expressão de n(k) em função de k diretamente.
>
> De onde veio este problema?
>
> []s,
> Claudio.
>
>
> 2018-04-10 18:11 GMT-03:00 Douglas Oliveira de Lima <
> profdouglaso.del...@gmail.com>:
>
>> Caros amigos , retomando o raciocinio, rs, estou com um problema um tanto
>> interessante que nao sei como fazer:
>>
>> Existe algum jeito de calcular o valor do somatório dos produtos
>> n(k).(101-k) onde k varia de 1 a 98 e n(k) é o número de divisores de k.
>>
>>
>> Qualquer ajuda será bem vinda.
>>
>>
>> Abraco do
>> Douglas Oliveira.
>>
>>
>> --
>> Esta mensagem foi verificada pelo sistema de antivírus e
>> acredita-se estar livre de perigo.
>
>
>
> --
> Esta mensagem foi verificada pelo sistema de antivírus e
> acredita-se estar livre de perigo.

-- 
Esta mensagem foi verificada pelo sistema de antiv�rus e
 acredita-se estar livre de perigo.



Re: [obm-l] Soma (k = 1, n) 1/P'(r_k) = 0

2018-04-13 Por tôpico Douglas Oliveira de Lima
Entao, sendo x^k=soma (i=1,...,n)(x_i)^k.P(x)/(x-x_i).P'(x_i) , é só
igualar os coeficientes de x^(n-1) e pronto, a identidade se torna ate mais
genérica

Soma (i= 1, n) (x_i)^k/P'(x_i) = 0

Obs: x_i sao raizes.

Abraco

Douglas Oliveira.




Em 8 de abr de 2018 20:50, "Artur Steiner" <artur.costa.stei...@gmail.com>
escreveu:

Seja P um polinômio complexo, de grau n >= 2, que tenha n raízes simples
r_1, ... r_n. Mostre que Soma (k = 1, n) 1/P'(r_k) = 0.

Para quem conhece um pouco de análise complexa, isto é corolário de um
resultado geral. Mas parece que pode ser provado sem análise complexa.

Artur Costa Steiner

-- 
Esta mensagem foi verificada pelo sistema de antivírus e
acredita-se estar livre de perigo.

-- 
Esta mensagem foi verificada pelo sistema de antiv�rus e
 acredita-se estar livre de perigo.



[obm-l] Geometria

2018-04-12 Por tôpico Douglas Oliveira de Lima
Caros amigos , tenho um problema bem legal e estou compartilhando. Ai vai:

Numa reta marcam-se os pontos A,B,C,D nesta ordem , e no mesmo semiplano
constroem-se os triângulos equiláteros ABP, BCQ e CDR de lados 5, 3 e x
respectivamente, sendo o angulo PQR igual a 120 graus, determine x.



Será que teria alguma construção bonita para solucionå-lo?

Abraco
Douglas Oliveira.

-- 
Esta mensagem foi verificada pelo sistema de antiv�rus e
 acredita-se estar livre de perigo.



[obm-l] Combinatória

2018-04-10 Por tôpico Douglas Oliveira de Lima
Caros amigos , retomando o raciocinio, rs, estou com um problema um tanto
interessante que nao sei como fazer:

Existe algum jeito de calcular o valor do somatório dos produtos
n(k).(101-k) onde k varia de 1 a 98 e n(k) é o número de divisores de k.


Qualquer ajuda será bem vinda.


Abraco do
Douglas Oliveira.

-- 
Esta mensagem foi verificada pelo sistema de antiv�rus e
 acredita-se estar livre de perigo.



Re: [obm-l] Geometria plana

2018-04-02 Por tôpico Douglas Oliveira de Lima
Da para fazer uma prova por absurdo.
Fica bom, suponha que a reta nao tangencia a circunferencia entao trace a
tangente e vai chegar em um absurdo.

Abraco
Douglas Oliveira.

Em seg, 2 de abr de 2018 11:14, Claudio Arconcher <arclaud...@hotmail.com>
escreveu:

> Bom dia caros colegas.
>
> Ponhamos ABCD o quadrado (o ponto A está no lado de baixo e à esquerda,
> segue-se o ponto B à direita, C e D estão no lado de acima fechando o
> circuito ABCD ).
>
> Ponhamos: AP=x e AQ=y, segue-se, QD=1-y e PB=1-x.
>
> Tracemos a circunferência de centro C e raio 1, ela tangencia AD em D e AB
> em B, agora seja M um ponto no quarto dessa circunferência interno ao
> quadrado ABCD e tracemos a tangente a ela por M, cortando AD em Q e AB em P
> ( serão, de fato os pontos esperados ), tem-se: QD=1-y = QM e PB=1 – x =
> PM, o perímetro do triângulo retângulo QAP é igual a 2. Reciprocamente se
> consideramos o triângulo AQP de perímetro 2 fixado antes o ponto M será o
> mesmo, todos esses triângulos são assim obtidos, com PQ tangente à
> circunferência em um ponto M com a propriedade descrita.
>
> Agora basta examinar as congruências dos triângulos retângulos CDQ e CMQ
> e, também, CBP e CMP, isso nos leva a concluir que o ângulo PCQ mede 45 º.
>
> Espero que o “coelhinho da Páscoa” concorde comigo.
>
> Abraço.
>
> Cláudio.
>
>
>
> *De:* owner-ob...@mat.puc-rio.br [mailto:owner-ob...@mat.puc-rio.br] *Em
> nome de *Douglas Oliveira de Lima
> *Enviada em:* domingo, 1 de abril de 2018 17:25
> *Para:* obm-l@mat.puc-rio.br
> *Assunto:* [obm-l] Geometria plana
>
>
>
> Olá amigos, pra quem gosta de geometria plana, compartilhando aqui uma
> questão do coelhinho da páscoa que achei legal.
>
>
>
> 1) Em um quadrado ABCD de lado unitário tomam-se os pontos P e Q sobre os
> lados AB e AD respectivamente, de modo que o perímetro do triângulo APQ
> seja igual a 2. Calcule a medida do ângulo  PCQ.
>
>
>
> Um abraço
>
>
>
> Douglas Oliveira.
>
>
> --
> Esta mensagem foi verificada pelo sistema de antiv�rus e
> acredita-se estar livre de perigo.
>
>
> <https://www.avast.com/sig-email?utm_medium=email_source=link_campaign=sig-email_content=emailclient>
>  Livre
> de vírus. www.avast.com
> <https://www.avast.com/sig-email?utm_medium=email_source=link_campaign=sig-email_content=emailclient>.
> <#m_3844324294932745576_DAB4FAD8-2DD7-40BB-A1B8-4E2AA1F9FDF2>
>
> --
> Esta mensagem foi verificada pelo sistema de antivírus e
> acredita-se estar livre de perigo.
>

-- 
Esta mensagem foi verificada pelo sistema de antiv�rus e
 acredita-se estar livre de perigo.



Re: [obm-l] Geometria plana

2018-04-02 Por tôpico Douglas Oliveira de Lima
Entao Claudio, eu pensei assim tb, mas a parte do reciprocamente, me deixa
incomodado, pois se o perimetro for 2 como provar que a circunferencia
tangencia em M.

Douglas Oliveira.

Em seg, 2 de abr de 2018 11:14, Claudio Arconcher <arclaud...@hotmail.com>
escreveu:

> Bom dia caros colegas.
>
> Ponhamos ABCD o quadrado (o ponto A está no lado de baixo e à esquerda,
> segue-se o ponto B à direita, C e D estão no lado de acima fechando o
> circuito ABCD ).
>
> Ponhamos: AP=x e AQ=y, segue-se, QD=1-y e PB=1-x.
>
> Tracemos a circunferência de centro C e raio 1, ela tangencia AD em D e AB
> em B, agora seja M um ponto no quarto dessa circunferência interno ao
> quadrado ABCD e tracemos a tangente a ela por M, cortando AD em Q e AB em P
> ( serão, de fato os pontos esperados ), tem-se: QD=1-y = QM e PB=1 – x =
> PM, o perímetro do triângulo retângulo QAP é igual a 2. Reciprocamente se
> consideramos o triângulo AQP de perímetro 2 fixado antes o ponto M será o
> mesmo, todos esses triângulos são assim obtidos, com PQ tangente à
> circunferência em um ponto M com a propriedade descrita.
>
> Agora basta examinar as congruências dos triângulos retângulos CDQ e CMQ
> e, também, CBP e CMP, isso nos leva a concluir que o ângulo PCQ mede 45 º.
>
> Espero que o “coelhinho da Páscoa” concorde comigo.
>
> Abraço.
>
> Cláudio.
>
>
>
> *De:* owner-ob...@mat.puc-rio.br [mailto:owner-ob...@mat.puc-rio.br] *Em
> nome de *Douglas Oliveira de Lima
> *Enviada em:* domingo, 1 de abril de 2018 17:25
> *Para:* obm-l@mat.puc-rio.br
> *Assunto:* [obm-l] Geometria plana
>
>
>
> Olá amigos, pra quem gosta de geometria plana, compartilhando aqui uma
> questão do coelhinho da páscoa que achei legal.
>
>
>
> 1) Em um quadrado ABCD de lado unitário tomam-se os pontos P e Q sobre os
> lados AB e AD respectivamente, de modo que o perímetro do triângulo APQ
> seja igual a 2. Calcule a medida do ângulo  PCQ.
>
>
>
> Um abraço
>
>
>
> Douglas Oliveira.
>
>
> --
> Esta mensagem foi verificada pelo sistema de antiv�rus e
> acredita-se estar livre de perigo.
>
>
> <https://www.avast.com/sig-email?utm_medium=email_source=link_campaign=sig-email_content=emailclient>
>  Livre
> de vírus. www.avast.com
> <https://www.avast.com/sig-email?utm_medium=email_source=link_campaign=sig-email_content=emailclient>.
> <#m_3844324294932745576_DAB4FAD8-2DD7-40BB-A1B8-4E2AA1F9FDF2>
>
> --
> Esta mensagem foi verificada pelo sistema de antivírus e
> acredita-se estar livre de perigo.
>

-- 
Esta mensagem foi verificada pelo sistema de antiv�rus e
 acredita-se estar livre de perigo.



[obm-l] Geometria plana

2018-04-01 Por tôpico Douglas Oliveira de Lima
Olá amigos, pra quem gosta de geometria plana, compartilhando aqui uma
questão do coelhinho da páscoa que achei legal.

1) Em um quadrado ABCD de lado unitário tomam-se os pontos P e Q sobre os
lados AB e AD respectivamente, de modo que o perímetro do triângulo APQ
seja igual a 2. Calcule a medida do ângulo  PCQ.

Um abraço

Douglas Oliveira.

-- 
Esta mensagem foi verificada pelo sistema de antiv�rus e
 acredita-se estar livre de perigo.



[obm-l] Functional equation(ajuda)

2018-03-31 Por tôpico Douglas Oliveira de Lima
Olá caros amigos, preciso de uma ajuda para resolver os seguintes problemas:

1) Uma função f:N*-->N* é tal que 0<=f(1)<204 e, para todo n>0, tem-se que
f(n+1)=(n/2004 +1/n)[f(n)]^2-(n^3)/2004 +1.
A quantidade de elementos da imagem de f que são números primos é:

2)Sejam u e v números reais tais que IuI<=3, IvI<=2. Determine o valor
mínimo de
f(u,v)=(u-v)^2+[((144-16u^2)^(1/2))/3 - (4-v^2)^(1/2)]^2.

Forte abraço.

Douglas Oliveira.

-- 
Esta mensagem foi verificada pelo sistema de antiv�rus e
 acredita-se estar livre de perigo.



Re: [obm-l] probleminhas de geometria

2018-03-28 Por tôpico Douglas Oliveira de Lima
Entao. acho que para qualquer circunferencia(concentrica ) sai usando
complexos, vamos ver,

O valor pedido será (w-Z1)(w-z1)+(w-Z2)(w-z2)+(w-Z3)(w-z3)=A, onde z1 é o
conjugado de Z1.

Podemos representar a circunferencia por modulo de w igual a r e o
triangulo equilatero por z^3-k^3=0 .

Assim o valor de A será 3r^2+3k^2-w(Z1+z1+Z2+z2+Z3+z3) logo A=3r^2+3k^2.

Pronto morreu.


Um abraco
 Douglas Oliveira.
Mas o valor de A será


Em 27 de mar de 2018 12:06, "Claudio Buffara" <claudio.buff...@gmail.com>
escreveu:

Achei estes dois bonitinhos:

1) Prove que, sendo P um ponto qualquer da circunferência inscrita a um
triângulo equilátero ABC, PA^2 + PB^2 + PC^2 é constante.
1A) Prove que isso vale para qualquer circunferência concêntrica com o
incírculo (tem uma demonstração legal para o circumcírculo usando o teorema
de Ptolomeu).


2) Um bolo tem a forma de um paralelepípedo retângulo de base quadrada e
tem cobertura no topo e nas quatro faces.
Mostre como dividir o bolo entre 7 pessoas de modo que cada um receba a
mesma quantidade de bolo e de cobertura.

Obs: a solução que envolve bater o bolo num liquidificador e dividir a
gororoba resultante em 7 partes de mesmo peso não é válida.

[]s,
Claudio.



-- 
Esta mensagem foi verificada pelo sistema de antivírus e
acredita-se estar livre de perigo.

-- 
Esta mensagem foi verificada pelo sistema de antiv�rus e
 acredita-se estar livre de perigo.



[obm-l] Re: [obm-l] Re: [obm-l] Teoria dos números

2018-03-19 Por tôpico Douglas Oliveira de Lima
E (-1,-1,2) e suas permutacoes.

Em 19 de mar de 2018 10:25, "Pedro José" <petroc...@gmail.com> escreveu:

> Bom dia!
>
> Poderia postar a solução? Não consegui achar nenhuma restrição para
> trabalhar num subconjunto  pequeno dos inteiros.
> Creio que vá ser apenas a trivial (0,0,1) e suas permutações.
>
> grato,
> PJMS
>
> Em 13 de março de 2018 20:19, Douglas Oliveira de Lima <
> profdouglaso.del...@gmail.com> escreveu:
>
>> Essa achei legal e estou postando.
>>
>> *Resolva nos inteiros a seguinte equação:  (x + y)(y + z)(z + x)/2 + (x +
>> y + z)3 = 1 – xyz* .
>>
>> Abraço do
>> Douglas Oliveira
>>
>> --
>> Esta mensagem foi verificada pelo sistema de antivírus e
>> acredita-se estar livre de perigo.
>
>
>
> --
> Esta mensagem foi verificada pelo sistema de antivírus e
> acredita-se estar livre de perigo.

-- 
Esta mensagem foi verificada pelo sistema de antiv�rus e
 acredita-se estar livre de perigo.



[obm-l] Teoria dos números

2018-03-13 Por tôpico Douglas Oliveira de Lima
Essa achei legal e estou postando.

*Resolva nos inteiros a seguinte equação:  (x + y)(y + z)(z + x)/2 + (x + y
+ z)3 = 1 – xyz* .

Abraço do
Douglas Oliveira

-- 
Esta mensagem foi verificada pelo sistema de antiv�rus e
 acredita-se estar livre de perigo.



[obm-l] Ajuda em geometria e álgebra.

2018-03-13 Por tôpico Douglas Oliveira de Lima
Olá amigos, não consigo fazer esse problema por construção, já fiz por lei
dos senos e
pelo geogebra e deu 18 graus.

Eis o problema:

6 Seja D um ponto sobre o lado BC de um triângulo ABC. Supondo que, AC=BD
e o ângulo  ADC=30 graus e ACB= 48 graus , determine  a medida do ângulo
ABC.



Qualquer ajuda será bem vinda.

Abraço do

Douglas Oliveira.

-- 
Esta mensagem foi verificada pelo sistema de antiv�rus e
 acredita-se estar livre de perigo.



[obm-l] Re: [obm-l] Re: [obm-l] Álgebra

2018-03-13 Por tôpico Douglas Oliveira de Lima
Valeu Ralph, Valeu Matheus , muito obrigado.
 Tinha mesmo pensado em algo semelhante, pensei da seguinte forma:
Quando a, b ou c são zero então a expressão dá zero, logo existe abc como
fator, daí,
a expressão remanescente de grau 2 assumiria a forma
x(a^2+b^2+c^2)+y(ab+ac+bc),
e substituindo valores acha-se x e y.

Mas de qualquer forma obrigadaço.

Forte abraço do
Douglas Oliveira.

Em 13 de março de 2018 19:16, Ralph Teixeira <ralp...@gmail.com> escreveu:

> Sim! Dá 80abc(a²+b²+c²)!
>
> ...
>
> ...
>
> Ah, você quer o JEITO... Huh... é bom, er... taquei no Scientific
> Workplace e mandei ele simplificar tudo desculpa. Talvez esteja até
> correto. :P
>
> Mas com a resposta em mãos alguém vai arrumar uma maneira bonita e
> criativa de chegar na mesmaresposta no braço, né? Né? Né?
>
> ...
>
> :D
>
> Abraços preguiçosos, Ralph.
>
> P.S.: Deve ter um jeito óbvio de ver que só os termos do tipo 3,1,1 ficam.
> Ah, sim: a expressão é ímpar em cada uma das variáveis, então todos os
> expoentes de cada variável têm que ser ímpares na resposta. Mas o polinômio
> é homogêneo, ou seja, a soma dos expoentes de cada termo é 5, então todos
> os termos são da forma a^m.b^n.c^p onde m+n+p=5 são ímpares. Acho que só
> 3+1+1 satisfaz ambas as condições? Como a expressão é invariante por
> permutação de variáveis, então só haverá um coeficiente, multiplicando os
> três monômios a^3bc, ab^3c, e abc^3, ou seja, já sei que tem que dar algo
> do tipo Kabc(a^2+b^2+c^2). Para achar K, taque a=b=c=1, e calibre K. Hm,
> acho que resolveu!
>
> 2018-03-13 18:51 GMT-03:00 Douglas Oliveira de Lima <
> profdouglaso.del...@gmail.com>:
>
>> Olá meus amigos, vocês conhecem um jeito bom de simplificar isso
>> (a+b+c)^5-(a-b+c)^5-(a+b-c)^5-(b+c-a)^5
>>
>> Abraços
>> Douglas Oliveira
>>
>> --
>> Esta mensagem foi verificada pelo sistema de antivírus e
>> acredita-se estar livre de perigo.
>
>
>
> --
> Esta mensagem foi verificada pelo sistema de antivírus e
> acredita-se estar livre de perigo.

-- 
Esta mensagem foi verificada pelo sistema de antiv�rus e
 acredita-se estar livre de perigo.



[obm-l] Álgebra

2018-03-13 Por tôpico Douglas Oliveira de Lima
Olá meus amigos, vocês conhecem um jeito bom de simplificar isso
(a+b+c)^5-(a-b+c)^5-(a+b-c)^5-(b+c-a)^5

Abraços
Douglas Oliveira

-- 
Esta mensagem foi verificada pelo sistema de antiv�rus e
 acredita-se estar livre de perigo.



[obm-l] Re: [obm-l] Problema de minimização

2018-03-11 Por tôpico Douglas Oliveira de Lima
Seja o quadrilátero ABCD cujas diagonais são AC e BD, e O o ponto de
intersecção das diagonais.
Seja também um ponto P em seu interior e as distâncias PA, PB, PC, PD,
temos por desigualdade triângular
que PA+PC>=AC e PB+PD>=BD. Claramente vemos que o ponto P coincide com o
ponto O quando a soma das diagonais
coincide com a igualdade. Desta forma o ponto procurado é o encontro das
diagonais.


Forte abraço.
Douglas Oliveira.

Em 10 de março de 2018 21:07, Claudio Buffara <claudio.buff...@gmail.com>
escreveu:

> Aqui vai um bonitinho que eu nunca tinha visto:
>
> Dado um quadrilátero convexo, determine o ponto cuja soma das distâncias
> aos vértices do quadrilátero é mínima.
>
> Interessante que quando a distância entre dois vértices adjacentes dados
> tende a zero (e o quadrilátero “tende” a um triângulo), o ponto de mínimo
> não parece tender ao ponto de Fermat do triângulo (exceto quando o
> triângulo tem um ângulo >= 120 graus.
>
> Abs,
> Claudio.
>
> Enviado do meu iPhone
> --
> Esta mensagem foi verificada pelo sistema de antivírus e
>  acredita-se estar livre de perigo.
>
>
> =
> Instru�ões para entrar na lista, sair da lista e usar a lista em
> http://www.mat.puc-rio.br/~obmlistas/obm-l.html
> =
>

-- 
Esta mensagem foi verificada pelo sistema de antiv�rus e
 acredita-se estar livre de perigo.



[obm-l] Re: [obm-l] Re: [obm-l] Re: [obm-l] Re: [obm-l] Triângulo quase Russo - 12º - 18º - 42º

2018-03-01 Por tôpico Douglas Oliveira de Lima
Na vdd acho que confundi esse problema com outro sinistro rs.

Ah mas ta valendo, pelo menos agora agente tem outro.

Abracos.

Em 1 de mar de 2018 11:41, "Jeferson Almir" 
escreveu:

> Opa !! Deu um valor legal. Eu tinha errado a resposta é 48º. Desculpem
>
> Em qui, 1 de mar de 2018 às 11:27, Jeferson Almir <
> jefersonram...@gmail.com> escreveu:
>
>> Eu coloquei no Geogebra e deu 48,71º. Deve ter algo errado
>>
>> Em qua, 28 de fev de 2018 às 21:46, Anderson Torres <
>> torres.anderson...@gmail.com> escreveu:
>>
>>> Em 28 de fevereiro de 2018 11:59, Claudio Buffara
>>>  escreveu:
>>> > Sugestão 1: usando régua e transferidor, desenhe uma figura tão grande
>>> e
>>> > precisa quanto puder (por exemplo, ocupando a maior parte de uma folha
>>> de
>>> > A4).
>>> > Daí, meça o ângulo EDB com o transferidor e obtenha uma conjectura.
>>> > Já será um progresso: ao invés de ter que determinar o valor do ângulo
>>> e
>>> > provar que seu raciocínio está correto, você precisará apenas provar
>>> sua
>>> > conjectura.
>>> >
>>> > Sugestão 2: Como 12 = 360/30, considere um polígono regular convexo de
>>> 30
>>> > lados inscrito numa circunferência de centro A e tal que B e C sejam
>>> > vértices (adjacentes) do polígono.
>>>
>>> Tô tentando resolver dessa forma, mas acredito ser mais promissor usar
>>> um 15-ágono em que
>>> os três vértices estão na circunferência circunscrita ao triângulo.
>>> Dessa forma é mais fácil ver certas simetrias.
>>> Por exemplo, as retas que definem os ângulos inferiores atingem
>>> meios-arcos interessantes. Daí fica mais
>>> fácil verificar algumas propriedades.
>>>
>>> > Considere os vértices P e Q do polígono tais que PAB, BAC e CAQ são
>>> ângulos
>>> > adjacentes (digamos com P, B, C, Q tomados no sentido anti-horário
>>> sobre a
>>> > circunferência), com PAB = 84 graus e CAQ = 36 graus. Prove que BQ
>>> > intersecta AC em D e CP intersecta AB em E.
>>> > Será que a reta suporte de DE intersecta a circunferência em pontos
>>> que são
>>> > vértices do polígono de 30 lados?
>>> > Aqui está outra situação em que um desenho bem feito (agora também com
>>> um
>>> > compasso) pode ajudar.
>>> > Ou então, se você usar Geogebra ou algum outro software de geometria
>>> > dinâmica...
>>> >
>>> > []s,
>>> > Claudio.
>>> >
>>> >
>>> > 2018-02-28 7:36 GMT-03:00 Jeferson Almir :
>>> >>
>>> >> Queria uma ajuda nesse problema de preferência por geometria
>>> sintética :)
>>> >>
>>> >> Seja um triângulo ABC isósceles de base BC sendo  A = 12º e os pontos
>>> E e
>>> >> D sobre AB e BC respectivamente tal que os ângulos ECB= 42º e DBC
>>> =18º.
>>> >> Calcule o ângulo EDB.
>>> >>
>>> >> --
>>> >> Esta mensagem foi verificada pelo sistema de antivírus e
>>> >> acredita-se estar livre de perigo.
>>> >
>>> >
>>> >
>>> > --
>>> > Esta mensagem foi verificada pelo sistema de antivírus e
>>> > acredita-se estar livre de perigo.
>>>
>>> --
>>> Esta mensagem foi verificada pelo sistema de antivírus e
>>>  acredita-se estar livre de perigo.
>>>
>>>
>>> 
>>> =
>>> Instru�ões para entrar na lista, sair da lista e usar a lista em
>>> http://www.mat.puc-rio.br/~obmlistas/obm-l.html
>>> 
>>> =
>>>
>>
> --
> Esta mensagem foi verificada pelo sistema de antivírus e
> acredita-se estar livre de perigo.

-- 
Esta mensagem foi verificada pelo sistema de antiv�rus e
 acredita-se estar livre de perigo.



[obm-l] Re: [obm-l] Re: [obm-l] Re: [obm-l] Re: [obm-l] Triângulo quase Russo - 12º - 18º - 42º

2018-03-01 Por tôpico Douglas Oliveira de Lima
Eis a solução, quem me apresentou esse problema pela primeira vez foi meu
professor da UERJ Paulo César em 2003 se não me engano..
E depois peguei a revista que tinha a resolução com um grande amigo que
faleceu "Gandhi" Antonio Luis dos Santos.

O link da solução é
http://www.mat.puc-rio.br/~obmlistas/obm-l.200505/msg00212.html

Já postado pelo Nicolau tem tempo.

Vale a pena ler a revista é realmente muito boa, fala a respeito de 53
triplas de inteiros que satisfazem esse triângulo.

Forte abraço do
Douglas Oliveira.

Em 1 de março de 2018 11:31, Jeferson Almir <jefersonram...@gmail.com>
escreveu:

> Opa !! Deu um valor legal. Eu tinha errado a resposta é 48º. Desculpem
>
> Em qui, 1 de mar de 2018 às 11:27, Jeferson Almir <
> jefersonram...@gmail.com> escreveu:
>
>> Eu coloquei no Geogebra e deu 48,71º. Deve ter algo errado
>>
>> Em qua, 28 de fev de 2018 às 21:46, Anderson Torres <
>> torres.anderson...@gmail.com> escreveu:
>>
>>> Em 28 de fevereiro de 2018 11:59, Claudio Buffara
>>> <claudio.buff...@gmail.com> escreveu:
>>> > Sugestão 1: usando régua e transferidor, desenhe uma figura tão grande
>>> e
>>> > precisa quanto puder (por exemplo, ocupando a maior parte de uma folha
>>> de
>>> > A4).
>>> > Daí, meça o ângulo EDB com o transferidor e obtenha uma conjectura.
>>> > Já será um progresso: ao invés de ter que determinar o valor do ângulo
>>> e
>>> > provar que seu raciocínio está correto, você precisará apenas provar
>>> sua
>>> > conjectura.
>>> >
>>> > Sugestão 2: Como 12 = 360/30, considere um polígono regular convexo de
>>> 30
>>> > lados inscrito numa circunferência de centro A e tal que B e C sejam
>>> > vértices (adjacentes) do polígono.
>>>
>>> Tô tentando resolver dessa forma, mas acredito ser mais promissor usar
>>> um 15-ágono em que
>>> os três vértices estão na circunferência circunscrita ao triângulo.
>>> Dessa forma é mais fácil ver certas simetrias.
>>> Por exemplo, as retas que definem os ângulos inferiores atingem
>>> meios-arcos interessantes. Daí fica mais
>>> fácil verificar algumas propriedades.
>>>
>>> > Considere os vértices P e Q do polígono tais que PAB, BAC e CAQ são
>>> ângulos
>>> > adjacentes (digamos com P, B, C, Q tomados no sentido anti-horário
>>> sobre a
>>> > circunferência), com PAB = 84 graus e CAQ = 36 graus. Prove que BQ
>>> > intersecta AC em D e CP intersecta AB em E.
>>> > Será que a reta suporte de DE intersecta a circunferência em pontos
>>> que são
>>> > vértices do polígono de 30 lados?
>>> > Aqui está outra situação em que um desenho bem feito (agora também com
>>> um
>>> > compasso) pode ajudar.
>>> > Ou então, se você usar Geogebra ou algum outro software de geometria
>>> > dinâmica...
>>> >
>>> > []s,
>>> > Claudio.
>>> >
>>> >
>>> > 2018-02-28 7:36 GMT-03:00 Jeferson Almir <jefersonram...@gmail.com>:
>>> >>
>>> >> Queria uma ajuda nesse problema de preferência por geometria
>>> sintética :)
>>> >>
>>> >> Seja um triângulo ABC isósceles de base BC sendo  A = 12º e os pontos
>>> E e
>>> >> D sobre AB e BC respectivamente tal que os ângulos ECB= 42º e DBC
>>> =18º.
>>> >> Calcule o ângulo EDB.
>>> >>
>>> >> --
>>> >> Esta mensagem foi verificada pelo sistema de antivírus e
>>> >> acredita-se estar livre de perigo.
>>> >
>>> >
>>> >
>>> > --
>>> > Esta mensagem foi verificada pelo sistema de antivírus e
>>> > acredita-se estar livre de perigo.
>>>
>>> --
>>> Esta mensagem foi verificada pelo sistema de antivírus e
>>>  acredita-se estar livre de perigo.
>>>
>>>
>>> 
>>> =
>>> Instru�ões para entrar na lista, sair da lista e usar a lista em
>>> http://www.mat.puc-rio.br/~obmlistas/obm-l.html
>>> 
>>> =
>>>
>>
> --
> Esta mensagem foi verificada pelo sistema de antivírus e
> acredita-se estar livre de perigo.
>

-- 
Esta mensagem foi verificada pelo sistema de antiv�rus e
 acredita-se estar livre de perigo.



Re: [obm-l] Como calcular?

2018-03-01 Por tôpico Douglas Oliveira de Lima
Então, esse problema é bem interessante, se eu não me engano,
ele tem sua origem com o matemático indiano Ramanujam, em um
de seus escritos.

Mas tem uma solução legal na dissertação do meu camarada Carlos Victor, do
PROFMAT,
veja: https://sca.profmat-sbm.org.br/sca_v2/get_tcc3.php?id=27919 , é o
problema de número 49.

Valeu forte abraço do
Douglas Oliveira.

Em 1 de março de 2018 10:08, Bernardo Freitas Paulo da Costa <
bernardo...@gmail.com> escreveu:

> 2018-03-01 0:56 GMT-03:00 Gabriel Tostes <gtos...@icloud.com>:
> > Define a sequencia A_(n+1)= [ (A_n)^2 - 1 ] / n (1)
> > Então A_2= sqrt(1+2A3)=sqrt(1+2(sqrt(1+3A4))... Realimentando sempre
> (substituindo A_n=sqrt(1+ nA_n+1)
> > vemos que A2 se iguala a x se lim n->oo da raiz 2^(n-2) de An é 0.
>
> Eu não entendi esta afirmação "A2 se iguala a x se lim ... = 0".  Como
> você mostra isso?  Além do mais, a sequência de raizes podia
> (podia...) tender a infinito.  Acho que também tem que mostrar que não
> é o caso.  Enfim, o que você escreveu (pode ser que você queira dizer
> outra coisa) é que "se o limite é zero, então A2 é finito", mas o que
> você precisa (para o argumento abaixo) é "se A2 é finito, então o
> limite é zero".
>
> > Seja An=n+1 + Bn. Bn outra sequencia.
>
> Realmente, essa transformação é mágica.  Eu chutei o limite (usando um
> computador) e daí calculei os outros termos, vi An = n+1, e fui provar
> que dava.  O que eu usei foi uma sequência dupla, T(n,m) = raiz(1 +
> n*raiz(1 + (n+1)*raiz(1 +  raiz(1 + m) ... ))).  Claro que
> T(n,m+1) > T(n,m), portanto lim T(n,m) existe (ou é infinito).  E para
> provar que não é infinito eu usei que o limite deveria dar T(n,inf) =
> n+1, e provei que T(n,m) < n+1 para todo m...
>
> > Então, de 1:
> > n+2+B_(n+1)=n+2 + 2Bn + ( Bn^2 + 2Bn)/n -> B_(n+1) >= 2Bn, uma inducao
> simples traz que:
> > Bn>=2^(n-2).B2
> > Entao o limite quando n vai para o infinito da raiz 2^(n-2) de An eh
> igual a B2, ou seja, B2=0 e
> > X= A2=3
>
> --
> Bernardo Freitas Paulo da Costa
>
> --
> Esta mensagem foi verificada pelo sistema de antivírus e
>  acredita-se estar livre de perigo.
>
>
> =
> Instru�ões para entrar na lista, sair da lista e usar a lista em
> http://www.mat.puc-rio.br/~obmlistas/obm-l.html
> =
>

-- 
Esta mensagem foi verificada pelo sistema de antiv�rus e
 acredita-se estar livre de perigo.



Re: [obm-l] determine all pair of integers (x,y) such that

2018-02-24 Por tôpico Douglas Oliveira de Lima
Então... como procuramos soluções inteiras, podemos ter também soluções
negativas.

1) Vamos lá, Se x<0 então 1+2^x+2^(2x+1) é inteiro somente se x=-1  logo
1+2^x+2^(2x+1)=2, mas 2 não é quadrado perfeito.

2) Se x=0 então 1+2^x+2^(2x+1)=4 então y=2 ou y=-2.

3)Se x>0 então 2^x+2^(2x+1)=2^x(1+2^(x+1))=(y-1)(y+1), como y é ímpar e
MDC(y-1, y+1)=1, temos que y-1=k2^(x-1) ou y+1=k2^(x-1), com k>0 e ímpar.

4) Se y-1=k2^(x-1) então 1+2^(x+1)=k+(k^2)2^(x-2) logo
2^(x-2)=(k-1)/(8-k^2), desta forma
8-k^2 I k-1, então pela desigualdade I 8-k^2 I<= I k-1 I, k=3 , 2^(x-2)=-2,
no qual não há soluções.

5) Se y+1=k2^(x-1), com a mesma analogia do passo 4 teremos 2^(x-2)=4, logo
x=4 e y=23 ou y=-23

Portanto as únicas soluções serão (0,2); (0,-2); (4,23); (4,-23).



Douglas Oliveira



Em 24 de fevereiro de 2018 09:47, Luís Lopes <qed_te...@hotmail.com>
escreveu:

> 1 + 2^x + 2^(2x+1) = y^2
>
>
> Sauda,c~oes,
>
>
> Recebi o problema acima de um outro grupo.
>
> Como resolver ?
>
>
> Abs,
>
> Luís
>
>
>
>
>
>
> --
> Esta mensagem foi verificada pelo sistema de antivírus e
> acredita-se estar livre de perigo.
>

-- 
Esta mensagem foi verificada pelo sistema de antiv�rus e
 acredita-se estar livre de perigo.



[obm-l] Teoria dos números :Solucões inteiras de uma equação

2017-12-15 Por tôpico Douglas Oliveira de Lima
Olá amigos , bom dia peço aos senhores uma ajuda no seguinte problema:

Dados a, b, k inteiros com k positivo e a equação x^2+axy+by^2=mt^k.

a) Determinar as condições de m para que a equação x^2+axy+by^2=mt^k tenha
soluções inteiras e encontrar as soluções quando existirem.

b) Examinar os casos k=2 e k=3.

Abraços do
DouglasOliveira.

-- 
Esta mensagem foi verificada pelo sistema de antiv�rus e
 acredita-se estar livre de perigo.



[obm-l] Probabilidade

2017-12-06 Por tôpico Douglas Oliveira de Lima
Caros amigos, preciso da ajuda dos senhores para confirmar um gabarito de
uma questão:

Eis a questão:

Num trem existem 280 animais, sendo 90 da fazenda Tampa, 110 da fazenda Boa
Vista, e 80 da fazenda Monte verde, se três dos animais fossem escolhidos
ao acaso entre os 280, qual a probabilidade de que cada um deles seja de
uma fazenda diferente?

Douglas Oliveira.

-- 
Esta mensagem foi verificada pelo sistema de antiv�rus e
 acredita-se estar livre de perigo.



Re: [obm-l] Probabilidade

2017-11-17 Por tôpico Douglas Oliveira de Lima
Valeu Pedro também achei esquisito.

Douglas Oliveira.

Em 17 de nov de 2017 16:49, "Pedro José" <petroc...@gmail.com> escreveu:

> Boa tarde!
>
> Não ficou claro o enunciado. Primeiramente cita que o lançamento é
> simultâneo, depois que Alfredo é o primeiro a jogar. tem uma vírgula
> seguida da expressão não há vencedor que não faz o menor sentido...
>
> Supondo que os lançamentos são intercalados. E que se uma pessoa atinge a
> soma 10 ganha e  o adversário não joga será:
>
> Chances favoráveis: (4,6); (6,4); (5,5) ==> 3 chances favoráveis e por
> conseguinte, 33 desfavoráveis.
>
> P10 = 1/12 e ~P10= 11/12.
>
> Para que Alfredo ganhe na segunda jogada será preciso: que Alfredo erre na
> primeira, *e* que Bernardo erre na primeira e que Alfredo acerte na
> segunda:
>
> P =(11/12)^2* (1/12)= 11^2/12^3.
>
> Supondo os lançamentos simultâneos, para o Alfredo ganhar na segunda além
> do s fatos do item anterior, ainda é necessário que o Bernardo erre a
> segunda.
>
> P* = P * 11/12 ==> P* = 11^3/12^4.
>
> Creio que seja isso. Saudações,
> PJMS
>
> Em 17 de novembro de 2017 15:03, Douglas Oliveira de Lima <
> profdouglaso.del...@gmail.com> escreveu:
>
>> Alfredo e Bernardo participam de um jogo participam de um jogo em que
>> cada um lança simultaneamente um par de dados até que um deles obtenha a
>> soma dos pontos das faces voltadas para cima igual a 10,momento em que a
>> disputa termina e o vencedor é o jogador que obteve essa soma 10,não há
>> vencedor. Se o Alfredo é o primeiro a jogar, qual é a probabilidade de que
>> ele seja o vencedor na segunda rodada (segundo lançamento de dados )
>>
>>
>>
>> --
>> Esta mensagem foi verificada pelo sistema de antivírus e
>> acredita-se estar livre de perigo.
>
>
>
> --
> Esta mensagem foi verificada pelo sistema de antivírus e
> acredita-se estar livre de perigo.

-- 
Esta mensagem foi verificada pelo sistema de antiv�rus e
 acredita-se estar livre de perigo.



[obm-l] Probabilidade

2017-11-17 Por tôpico Douglas Oliveira de Lima
Alfredo e Bernardo participam de um jogo participam de um jogo em que cada
um lança simultaneamente um par de dados até que um deles obtenha a soma
dos pontos das faces voltadas para cima igual a 10,momento em que a disputa
termina e o vencedor é o jogador que obteve essa soma 10,não há vencedor.
Se o Alfredo é o primeiro a jogar, qual é a probabilidade de que ele seja o
vencedor na segunda rodada (segundo lançamento de dados )

-- 
Esta mensagem foi verificada pelo sistema de antiv�rus e
 acredita-se estar livre de perigo.



[obm-l]

2017-10-18 Por tôpico Douglas Oliveira de Lima
Ola amigos, gostaria de uma ajuda  no seguinte problema:
Quem é maior? S=1/a+1/b+1/c ou t=a^(1/2)+b^(1/2)+c^(1/2) onde a, b e c sao
lados de um.triangulo e abc=1.

Obrigado.

-- 
Esta mensagem foi verificada pelo sistema de antiv�rus e
 acredita-se estar livre de perigo.



Re: [obm-l] soma de tan^2

2017-09-16 Por tôpico Douglas Oliveira de Lima
Eu resolvi esse problema em 2014 aqui na lista olhe
https://www.mail-archive.com/obm-l@mat.puc-rio.br/msg52281.html

Abraços.

Em 16 de set de 2017 13:23, "Carlos Gomes"  escreveu:

Olá Luis...lembro desse problema ...ele foi publicado na Mathematical
excalibur ha alguns anos https://www.math.ust.hk/excalibur/

A resposta é C(90,2)= 4005, se não me falha a memória...usa relações de
Girard num "polinômio esperto"...vou tenter ver se lembro a solução...se
lembrar ponha aqui!

Abraço, Cgomes.

Em 16 de setembro de 2017 10:48, Luís Lopes 
escreveu:

> Sauda,c~oes,
>
>
> Bom dia.
>
>
> Me mandaram a seguinte questão:
>
>
> (1) Seja S = tan²(1º) + tan²(3º) + tan²(5º) + ... + tan²(89º), calcule o
> valor de S.
>
> Como resolver ? Obrigado.
>
>
> Abs,
>
> Luís
>
>
>
> --
> Esta mensagem foi verificada pelo sistema de antivírus e
> acredita-se estar livre de perigo.
>


-- 
Esta mensagem foi verificada pelo sistema de antivírus e
acredita-se estar livre de perigo.

-- 
Esta mensagem foi verificada pelo sistema de antiv�rus e
 acredita-se estar livre de perigo.



[obm-l] Re: [obm-l] Re: [obm-l] Problema difícil.

2017-09-13 Por tôpico Douglas Oliveira de Lima
Então Bernardo, eu pensei numa parada mas não tenho certeza , pensei que os
números 997,998,999,...,1994 Não poderiam ocupar as posições de 1 a 1997,
logo pelo menos um deles ocuparia uma posição não inferior a 998, aí pensei
no 997.998=995006.

Em 12 de set de 2017 18:39, "Bernardo Freitas Paulo da Costa" <
bernardo...@gmail.com> escreveu:

> 2017-09-12 17:51 GMT-03:00 Douglas Oliveira de Lima
> <profdouglaso.del...@gmail.com>:
> > Considere a sequência de números 1,2,3,4,5,...,2017.
> > E uma certa ordenação deles a1, a2, a3, ..., a2017.
> > Agora multiplique respectivamente os números das duas sequencias
> > determinando assim uma nova sequência 1.a1, 2.a2, 3.a3, ..., 2017.a2017.
> >
> > Qual o menor valor que o maior produto da última sequência pode assumir?
>
> Esse problema não é tão difícil quanto parece.  O que você tentou fazer?
>
> Abraços,
> --
> Bernardo Freitas Paulo da Costa
>
> --
> Esta mensagem foi verificada pelo sistema de antivírus e
>  acredita-se estar livre de perigo.
>
>
> =
> Instru�ões para entrar na lista, sair da lista e usar a lista em
> http://www.mat.puc-rio.br/~obmlistas/obm-l.html
> =
>

-- 
Esta mensagem foi verificada pelo sistema de antiv�rus e
 acredita-se estar livre de perigo.



[obm-l] Problema difícil.

2017-09-12 Por tôpico Douglas Oliveira de Lima
Considere a sequência de números 1,2,3,4,5,...,2017.
E uma certa ordenação deles a1, a2, a3, ..., a2017.
Agora multiplique respectivamente os números das duas sequencias
determinando assim uma nova sequência 1.a1, 2.a2, 3.a3, ..., 2017.a2017.

Qual o menor valor que o maior produto da última sequência pode assumir?

Douglas Oliveira.

-- 
Esta mensagem foi verificada pelo sistema de antiv�rus e
 acredita-se estar livre de perigo.



Re: [obm-l] Fibonacci teoria dos numeros

2017-09-05 Por tôpico Douglas Oliveira de Lima
O problema caiu na olimpíada de matemática do Rio de Janeiro se não me
engano em 1999 ou 1998.

Em 5 de set de 2017 17:52, "Pedro José" <petroc...@gmail.com> escreveu:

> Boa tarde!
>
> O programa comera o F_28830 que é igual a zero.
> Desconsiderar o exposto anteriormente.
>
> Em 5 de setembro de 2017 16:25, Pedro José <petroc...@gmail.com> escreveu:
>
>> Boa tarde!
>>
>> Douglas,
>>
>> esse problema você viu em algum local ou foi uma conjectura sua?
>>
>> Fiz em computador a sequência de Fibonacci mod 29791, o o F_14 = 377
>> mod29791 e F_15 = 610 mod2971 e F_28844 = 377 mod29791 e F_28845 = 610
>> mod29791, o que caracteriza que haverá um padrão de repetição na geração
>> dos números de Fibonacci mod 29791. E como não tem um elemento F_i = 0 mod
>> para i <28844, não haverá mais nenhum termo F_j = 0. Pois para qualquer j
>> >= 28844 haverá um i < 28844, onde F_j = F_i <>0 mod 29791, logo 29791, não
>> tem um único múltiplo na sequência de Fibonacci e portanto, a hipótese é
>> falsa.
>>
>> Deve ter um modo mais elegante para mostrar que a proposição é fasla.
>>
>> Sds,
>> PJMS
>>
>>
>>
>>
>>
>>
>> Em 5 de setembro de 2017 10:16, Pedro José <petroc...@gmail.com>
>> escreveu:
>>
>>> Bom dia!
>>>
>>> Eu pensei que entendera, porém, os números não são sequenciais.
>>> Se nós tivermos dois números consecutivos F_j e F_j+1  congruentes
>>> módulo m, pela lei de geração da sequencia de Fibonacci teremos que F_j-1 =
>>> 0 módulo m.
>>> O enunciado deveria ter uma restrição ... existe um número de Fibonacci, não
>>> nulo, que é múltiplo n, para evitar a corrente que considera zero como
>>> o primeiro termo da sequencia, pois, aí ficaria elementar a solução.
>>> Acho que o caminho é provar que como F_1 = F_2  ==> que para algum j
>>> : F_j = F_j+1 módulo m o que leva a F_j-1= 0 módulo m.
>>> Mas por casa de pombo só não daria, por exemplo, se consideramos A_1 = 3
>>> e A_2 = 7 e Ai = A_i-1 + A_i-2, não haveria um número múltiplo de 8.
>>> A sequência mod 8 ficaria: 3, 7, 2, 1, 3, 4, 7, 3, 2, 5, 7, 4, 3, 7, 2,
>>> 1, 3, 4, 7, 3, 2, 5, 7, 4...
>>>
>>> Saudações,
>>> PJMS
>>>
>>>
>>> Em 4 de setembro de 2017 16:48, Pedro José <petroc...@gmail.com>
>>> escreveu:
>>>
>>>> Boa tarde!
>>>>
>>>> Nehab,
>>>>
>>>> não consegui entender o restante da solução, mas ele usou o sinal de
>>>> igual para congruência por comodidade de edição, e até pela lei de formação
>>>> da sequência, só o segundo e terceiro termos são iguais, quando se admite
>>>> que comece de zero, ou os dois primeiros, para a corrente que não considera
>>>> como o primeiro termo da sequencia..
>>>> Por exemplo, 13 = 23 mod 10 mas (13, 23) = 1. Portanto, não fere o
>>>> princípio de que dois números consecutivos na sequência de Fibonacci sejam
>>>> primos entre si.
>>>> Até aí captei e entendi, pelo princípio da casa de pombos. Estou
>>>> tentando entender o restante.
>>>>
>>>> Saudações,
>>>> PJMS
>>>>
>>>> Em 4 de setembro de 2017 14:53, Carlos Nehab <carlos.ne...@gmail.com>
>>>> escreveu:
>>>>
>>>>> Oi, Douglas.
>>>>>
>>>>> Acho que o mdc entre Fibbonaccis consecutivos é sempre 1...
>>>>>
>>>>> Nehab
>>>>>
>>>>>
>>>>> <https://www.avast.com/sig-email?utm_medium=email_source=link_campaign=sig-email_content=webmail>
>>>>>  Livre
>>>>> de vírus. www.avast.com
>>>>> <https://www.avast.com/sig-email?utm_medium=email_source=link_campaign=sig-email_content=webmail>.
>>>>>
>>>>> <#m_-2219119211184066607_m_-112899321461919009_m_4365848760417512581_m_408333944165922029_m_4928629599140568768_m_2006884623661450834_DAB4FAD8-2DD7-40BB-A1B8-4E2AA1F9FDF2>
>>>>>
>>>>> Em 4 de setembro de 2017 07:24, Anderson Torres <
>>>>> torres.anderson...@gmail.com> escreveu:
>>>>>
>>>>>> Em 31 de agosto de 2017 16:30, Douglas Oliveira de Lima
>>>>>> <profdouglaso.del...@gmail.com> escreveu:
>>>>>> > Olá, como posso mostrar que para algum inteiro e positivo n, existe
>>

[obm-l] Fibonacci teoria dos numeros

2017-08-31 Por tôpico Douglas Oliveira de Lima
Olá, como posso mostrar que para algum inteiro e positivo n, existe um
número de Fibonacci que é múltiplo de n?

Douglas Oliveira.

-- 
Esta mensagem foi verificada pelo sistema de antiv�rus e
 acredita-se estar livre de perigo.



[obm-l] Desigualdades

2017-08-14 Por tôpico Douglas Oliveira de Lima
Como posso prova para x,y,z positivos que
x^3+y^3+z^3+3xyz>=xy(x+y)+xz(x+z)+yz(y+z).

Douglas Oliveira .

-- 
Esta mensagem foi verificada pelo sistema de antiv�rus e
 acredita-se estar livre de perigo.



[obm-l] Teoria dos números

2017-08-14 Por tôpico Douglas Oliveira de Lima
Como posso mostrar que a sequência 1+1/2+1/3+1/4+1/5+...+1/n não é um
inteiro para n>1.

Forte abraço

Douglas Oliveira.

-- 
Esta mensagem foi verificada pelo sistema de antiv�rus e
 acredita-se estar livre de perigo.



[obm-l] Função máximo inteiro

2017-07-28 Por tôpico Douglas Oliveira de Lima
Quantas soluções positivas e inteiras possui a equação [n/10]=[n/11]+1 onde
[x] é o maior inteiro que não supera x.

Att.
Douglas Oliveira de Lima.

-- 
Esta mensagem foi verificada pelo sistema de antiv�rus e
 acredita-se estar livre de perigo.



Re: [obm-l] Geometria plana

2017-07-13 Por tôpico Douglas Oliveira de Lima
Eu fiz algo parecido , também cheguei na mesma resposta, eu cheguei na
expressão (m+n-n^2-m^2)/(m+n)(2-m-n) e tinha que maximizar isso com m e n
entre zero e um.

Obrigado.

Douglas Oliveira.

Em 12 de jul de 2017 4:10 PM, "Pedro José" <petroc...@gmail.com> escreveu:

> Boa tarde!
>
> Só faltaram as definições de a e b, a é a medida do segmento BF e b a do
> segmento CG.
>
> Desculpem-me,
> PJMS
>
> Em 12 de julho de 2017 09:08, Pedro José <petroc...@gmail.com> escreveu:
>
>> Bom dia!
>>
>> Estava indo pelo caminho errado, derivadas parciais.
>>
>> x + y = ab/(a+b) + (1-a) (1-b) / (2-(a+b)) = ((a+b) - (a^2+b^2))/ (2(a+b)
>> - (a+b)^2)
>>
>> Agora ficou fácil, basta mostrar que 2(a^2+b^2) >= (a+b)^2, o que implica
>> em x + y <= 0,5 e S(PFQG) <= 1/4
>> Mas por Cauchy-Shwarz fica clara a desigualdade e o caso particular da
>> igualdade só se dá para a = b.
>> Portanto, a=b é a condição e a área máxima é 1/4. Atentar que a e b não
>> podem assumir os valores 0 ou 1.
>>
>> Saudações,
>> PJMS
>>
>> Em 11 de julho de 2017 20:50, Pedro José <petroc...@gmail.com> escreveu:
>>
>>> Boa noite!
>>>
>>> Não consegui por completo, mas a solução é 1/4 e vale para BF=CG . BF<>0
>>> e BF <>1
>>>
>>> S(PFQG) =  S(FCD) - S(QCG) - S(PGD) ==> S(PFQG) = 1/2 - S(QCG) - S(PGD)
>>> (i)
>>>
>>> S (AGD) + S(BCG) = CG/2 +GD/2 = 1/2
>>>
>>> S(QCG) + S(PGD) + S(APD) + S(BCQ) = S (AGD) + S(BCG) = 1/2 (ii)
>>>
>>> por (i), se S(PFQG) é máximo então S(QCG) + S(PGD) é mínimo.
>>>
>>> por (ii) se S(QCG) + S(PGD) é mínimo, então S(APD) + S(BCQ) é máximo.
>>> (iii)
>>>
>>> seja x a medida da altura do triângulo BCQ, relativo ao vértice Q e y a
>>> altura do triângulo APD, relativa à P, de (iii) temos que x+ y deve ser
>>> máximo.
>>>
>>> x = ab/(a+b) e y = (1-a) (1-b) / (2-(a+b)), onde x é a medida de BF e y
>>> a medida de CG.
>>> É fácil mostrar que quando a=b ==> x+ y = 1/2 e S(PFQG) = 1/4.
>>>
>>> Difícil, pelo menos para mim, é mostrar que x + y < 0,5, quando a<>b e
>>> por conseguinte S(PFQG) < 1/4.
>>>
>>> Morri na praia.
>>>
>>> Saudações,
>>> PJMS
>>>
>>>
>>> Em 10 de julho de 2017 10:48, Douglas Oliveira de Lima <
>>> profdouglaso.del...@gmail.com> escreveu:
>>>
>>>> Sejam F e G pontos sobre AB e CD de um quadrado unitário ABCD. AG e DF
>>>> se interceptam em P,
>>>> e CF e BG se interceptam em Q. Determinar a posição dos pontos F e G
>>>> para que o quadrilátero PFQG tenha área máxima.
>>>>
>>>> Douglas Oliveira.
>>>>
>>>> --
>>>> Esta mensagem foi verificada pelo sistema de antivírus e
>>>> acredita-se estar livre de perigo.
>>>
>>>
>>>
>>
>
> --
> Esta mensagem foi verificada pelo sistema de antivírus e
> acredita-se estar livre de perigo.

-- 
Esta mensagem foi verificada pelo sistema de antiv�rus e
 acredita-se estar livre de perigo.



[obm-l] Problema de função elementar

2017-07-13 Por tôpico Douglas Oliveira de Lima
Seja F uma função crescente definida para todo número real x, 0<=x<=1, tal
que

a)  F(0)=0

b)  F(x/3)=F(x)/2

c)  F(1-x)=1-F(x)

Encontrar F(21/2017).


Douglas Oliveira

-- 
Esta mensagem foi verificada pelo sistema de antiv�rus e
 acredita-se estar livre de perigo.



[obm-l] Algebra (Polinomios)

2017-07-10 Por tôpico Douglas Oliveira de Lima
Encontrar o resto da divisão do polinomio (x^2+x+1)^40 por (x+1)^3.

Obs: Sem usar derivadas.

Douglas Oliveira.

-- 
Esta mensagem foi verificada pelo sistema de antiv�rus e
 acredita-se estar livre de perigo.



[obm-l] Álgebra (Equação funcional)

2017-07-10 Por tôpico Douglas Oliveira de Lima
Encontrar todas as funções f(x), definida nos reais, tais que

1) f(1)=1
2) f(x_1+x_2)=f(x_1)+f(x_2)
3) f(1/x)=(1/x^2).f(x), para x diferente de zero..

Douglas Oliveira

-- 
Esta mensagem foi verificada pelo sistema de antiv�rus e
 acredita-se estar livre de perigo.



Re: [obm-l] Somas iguais

2017-07-10 Por tôpico Douglas Oliveira de Lima
Queria propor um problema em cima desse, fiquei pensando que realmente é
possível de dividir em dois subgrupos,
a pergunta seria:

De quantas formas é possível dividir em dois subgrupos?

Douglas Oliveira.

Em 9 de julho de 2017 20:04, Vanderlei Nemitz <vanderma...@gmail.com>
escreveu:

> Obrigado, Pedro!
> Acho que ficou claro, sim!
>
> Em 8 de jul de 2017 3:51 PM, "Pedro Soares" <pedrosoares...@gmail.com>
> escreveu:
>
>> Desculpe se ficou mal escrito* heheh
>>
>>
>> <http://www.avg.com/email-signature?utm_medium=email_source=link_campaign=sig-email_content=webmail>
>>  Virus-free.
>> www.avg.com
>> <http://www.avg.com/email-signature?utm_medium=email_source=link_campaign=sig-email_content=webmail>
>> <#m_8448995251092151276_m_-8671497293299101645_DAB4FAD8-2DD7-40BB-A1B8-4E2AA1F9FDF2>
>>
>> 2017-07-08 15:26 GMT-03:00 Pedro Soares <pedrosoares...@gmail.com>:
>>
>>> Para a soma de n números naturais ser par essa sequência deve possuir um
>>> número par de números impares. Logo, se está se somando de 1 a n e a soma é
>>> par  para n = 2k - 1 ou n = 2k onde k é multiplo de 2( se k for impar
>>> teremos um número impar de números impares na soma).
>>> O caso em que n=2k é trivial, pode-se pegar os extremos da soma e
>>> colocar em um subgrupo, os próximos extremos colocar no outro subgrupo e
>>> repetir essa ação k/2 vezes( lembre-se que k é multiplo de 2, então podemos
>>> fazer isso).
>>> Para n = 2k - 1 primeiro olhe para k = 2, claramente podemos separar nos
>>> subgrupos {1,2} e {3} que possuem a mesma soma.
>>> Agora suponha que vale para k = j, vamos provar que vale para k = n + 2
>>> por indução.
>>> A soma para n = 2( k + 2 ) + 1 é igual a soma para n = 2k( que vamos
>>> chamar de S(n) ) mais quatro termos consecutivos ( n+1, n+2, n+3, n+4).
>>> S(n) já sabemos dividir em subgrupos de igual soma por hipótese. Além
>>> disso, podemos alocar os termos faltantes usando a mesma estratégia usada
>>> para o caso n=2k( os termos n+1 e n+4 vão para um subgrupo e os termos n+2
>>> e n+3 vão para o outro). Logo, se vale para k = j vale k = j + 2. Como vale
>>> para k = 2 vale para todo multiplo de 2.
>>> Como já provamos para os dois casos em que separamos isso conclui nossa
>>> prova :)
>>>
>>> Desculpe se ficou mau escrito, digitei conforme fui pensando
>>>
>>>
>>> On Saturday, 8 July 2017, Vanderlei Nemitz <vanderma...@gmail.com>
>>> wrote:
>>>
>>>> Bom dia!
>>>> Gostaria de saber se alguém tem uma solução para esse problema:
>>>>
>>>> *Mostre que se a soma dos números de 1 até n é par, então é possível
>>>> separar os números de 1 até n em dois subgrupos de números de igual soma.*
>>>>
>>>> Muito obrigado!
>>>>
>>>> Vanderlei
>>>>
>>>> --
>>>> Esta mensagem foi verificada pelo sistema de antivírus e
>>>> acredita-se estar livre de perigo.
>>>
>>>
>>
>> --
>> Esta mensagem foi verificada pelo sistema de antivírus e
>> acredita-se estar livre de perigo.
>
>
> --
> Esta mensagem foi verificada pelo sistema de antivírus e
> acredita-se estar livre de perigo.
>

-- 
Esta mensagem foi verificada pelo sistema de antiv�rus e
 acredita-se estar livre de perigo.



[obm-l] Geometria plana

2017-07-10 Por tôpico Douglas Oliveira de Lima
Sejam F e G pontos sobre AB e CD de um quadrado unitário ABCD. AG e DF se
interceptam em P,
e CF e BG se interceptam em Q. Determinar a posição dos pontos F e G para
que o quadrilátero PFQG tenha área máxima.

Douglas Oliveira.

-- 
Esta mensagem foi verificada pelo sistema de antiv�rus e
 acredita-se estar livre de perigo.



[obm-l] Geometria plana

2017-07-08 Por tôpico Douglas Oliveira de Lima
Num triângulo equilátero ABC, as cevianas BD e CE se encontram em P, se a
área do triângulo BCP é igual a área do quadrilátero ADPE ,  determine o
ângulo BPC.

Douglas Oliveira.

-- 
Esta mensagem foi verificada pelo sistema de antiv�rus e
 acredita-se estar livre de perigo.



[obm-l] Re: [obm-l] Re: [obm-l] Teoria dos números

2017-07-06 Por tôpico Douglas Oliveira de Lima
Opa , sim, é a•b•c

Em 6 de jul de 2017 11:14 PM, "Carlos Nehab" <carlos.ne...@gmail.com>
escreveu:

> Oi, Douglas,
>
> Esse "abc" é a x b x c (produto) ou o inteiro de algarismos a, b e c
> (100a+10b+c)?
>
> Abs
> Nehab
>
>
> <https://www.avast.com/sig-email?utm_medium=email_source=link_campaign=sig-email_content=webmail>
>  Livre
> de vírus. www.avast.com
> <https://www.avast.com/sig-email?utm_medium=email_source=link_campaign=sig-email_content=webmail>.
> <#m_1267597801263667645_DAB4FAD8-2DD7-40BB-A1B8-4E2AA1F9FDF2>
>
> Em 6 de julho de 2017 14:03, Douglas Oliveira de Lima <
> profdouglaso.del...@gmail.com> escreveu:
>
>> Encontrar todos os inteiros positivos a,b e c tais que a^b+b^c=abc.
>>
>> --
>> Esta mensagem foi verificada pelo sistema de antivírus e
>> acredita-se estar livre de perigo.
>
>
>
> --
> Esta mensagem foi verificada pelo sistema de antivírus e
> acredita-se estar livre de perigo.

-- 
Esta mensagem foi verificada pelo sistema de antiv�rus e
 acredita-se estar livre de perigo.



[obm-l] Teoria dos números

2017-07-06 Por tôpico Douglas Oliveira de Lima
Encontrar todos os inteiros positivos a,b e c tais que a^b+b^c=abc.

-- 
Esta mensagem foi verificada pelo sistema de antiv�rus e
 acredita-se estar livre de perigo.



Re: [obm-l] Geometria plana (Ajuda)

2017-06-28 Por tôpico Douglas Oliveira de Lima
Opa desculpe, CF é ceviana que passa por P.

Em 28 de jun de 2017 11:05 AM, "Pedro José" <petroc...@gmail.com> escreveu:

> Bom dia!
>
> O ponto F não foi definido, mas foram definidas duas medidas de ângulos
> aos quais o ponto F pertence: BCF=20 graus e FCA=40 graus.
> Não faltou definir o ponto F?
>
> Sds,
> PJMS
>
> Em 28 de junho de 2017 09:15, Douglas Oliveira de Lima <
> profdouglaso.del...@gmail.com> escreveu:
>
>> Olá meus amigos preciso de uma ajuda pra resolver a seguinte questão:
>>
>> Num triângulo ABC , tracam-se as cevianas AD e BE, que se encontram no
>> ponto P, tal que BAD= 10 graus, DAC=70 graus, BCF=20 graus e FCA=40 graus,
>> traçando a ceviana BE que passa por P e o segmento de reta que une os
>> pontos E e M, sendo M ponto médio de BC, determinar o ângulo CME.
>>
>> GRATO!!
>> Douglas Oliveira.
>>
>> --
>> Esta mensagem foi verificada pelo sistema de antivírus e
>> acredita-se estar livre de perigo.
>
>
>
> --
> Esta mensagem foi verificada pelo sistema de antivírus e
> acredita-se estar livre de perigo.

-- 
Esta mensagem foi verificada pelo sistema de antiv�rus e
 acredita-se estar livre de perigo.



[obm-l] Geometria plana (Ajuda)

2017-06-28 Por tôpico Douglas Oliveira de Lima
Olá meus amigos preciso de uma ajuda pra resolver a seguinte questão:

Num triângulo ABC , tracam-se as cevianas AD e BE, que se encontram no
ponto P, tal que BAD= 10 graus, DAC=70 graus, BCF=20 graus e FCA=40 graus,
traçando a ceviana BE que passa por P e o segmento de reta que une os
pontos E e M, sendo M ponto médio de BC, determinar o ângulo CME.

GRATO!!
Douglas Oliveira.

-- 
Esta mensagem foi verificada pelo sistema de antiv�rus e
 acredita-se estar livre de perigo.



Re: [obm-l] Radicais

2017-06-04 Por tôpico Douglas Oliveira de Lima
Opa amigo, o radical do Indiano Ramanujam, baixe um arquivo do Carlos
Victor , muito bom tem esse problema resolvido e vários outros.
Segue o link
http://cursos.ufrrj.br/posgraduacao/profmat/dissertacoes/dissertacoe/

Um abraço
Douglas Oliveira.

Em 4 de jun de 2017 3:19 PM, "Pedro Júnior" <pedromatematic...@gmail.com>
escreveu:

> Olá pessoal, vocês poderiam me ajudar a solucionar o problema abaixo? Já
> vi alguns bem parecidos, mas esse está me pegando...
>
> Raiz (1+2Raiz(1+3Raiz(1+4Raiz(1+...= ?
>
> Desde já agradeço
>
> --
> Esta mensagem foi verificada pelo sistema de antivírus e
> acredita-se estar livre de perigo.

-- 
Esta mensagem foi verificada pelo sistema de antiv�rus e
 acredita-se estar livre de perigo.



[obm-l] Sistema.

2017-06-04 Por tôpico Douglas Oliveira de Lima
Olá amigos, podem me dar uma ajuda no seguinte problema:
{a/b + c/d = -1, a^2 + c^2 = 1, b^2 + d^2 = 1, b^3/a + d^3/c = x},
encontrar x.

Abraços
Douglas Oliveira.

-- 
Esta mensagem foi verificada pelo sistema de antiv�rus e
 acredita-se estar livre de perigo.



Re: [obm-l] Desigualdade

2017-05-28 Por tôpico Douglas Oliveira de Lima
A_1=3

Em 28 de mai de 2017 12:44 PM, "Esdras Muniz" <esdrasmunizm...@gmail.com>
escreveu:

> Se vc colocar a1 igual a 0, 1 ou 2 vai ver queisso não é verdade. Acho que
> é verdade se |a1|>e.
>
> Em 28 de mai de 2017 11:58, "Douglas Oliveira de Lima" <
> profdouglaso.del...@gmail.com> escreveu:
>
> Então amigos, eu tive uma idéia mas não estou conseguindo concluir, vamos
> lá:
>
> Montei uma sequência e fiz a_1=3, e assim 
> [2^(1/2)].[3^(1/4)].[4^(1/8)].[5^(1/16)]...<3
> se, e somente se,
> [3^(1/4)].[4^(1/8)].[5^(1/16)]...<(3^2)/2, portanto a_2=(3^2)/2, e assim
> sucessivamente escrevi a sequência
> a_n=(a_(n-1)^2)/n, e usei um "leminha" para resolver, que é (a_n)^2>n+1, e
> assim sai fácil, só não consegui escrever
> a prova desse lema.
>
> Mas com ele sai bem facil, pois se  (a_n)^2>n+1, então (a_n)>(n+1)^(1/2),
> logo  (a_(n-1)^2)/n>(n+1)^(1/2), ou seja,
> a_(n-1)>n^(1/2)(n+1)^(1/4), ., 
> a_1>[2^(1/2)].[3^(1/4)].[4^(1/8)].[5^(1/16)]...,
> e como a_1=3, está provado.
>
> Peço a ajuda de vocÊs para provar o lema.
>
>  Lema:
> Considere a sequência a_n=(a_(n-1)^2)/n, onde a_1=3 então (a_n)^2>n+1.
>
>
>
> Douglas Oliveira
>
>
>
>
> Em 27 de maio de 2017 18:10, Esdras Muniz <esdrasmunizm...@gmail.com>
> escreveu:
>
>> Solução muito boa.
>>
>> Em 27 de mai de 2017 00:37, "Gabriel Tostes" <gtos...@icloud.com>
>> escreveu:
>>
>>> Tira ln, esse produto vai ser:
>>> Sum{n>=1} ln(n+1)/(2^n) = M
>>>
>>> Bora escrever M de outro jeito:
>>>
>>> M= ln(2) + [ln(3)-ln(2)]/2 + [ln(4)-ln(3)]/2^2 + ...
>>>
>>> M= Sum{n>=1} (ln(n+1)-ln(n))/2^(n-1)
>>>
>>> Como ln(n+1)-ln(n)=ln(1+1/n)<1/n
>>>
>>> M=2} 1/n.2^(n-1) = L + ln(2)
>>>
>>> Para achar L considere:
>>> 1/(1-x)= 1+x^2+x^3+...
>>>
>>> Integrando essa expressao temos que -(1/x).ln(1-x)= 1+x/2+x^2/3+...
>>> Substituindo x=1/2 achamos que L=2ln(2)-1
>>> E entao
>>> M< 3ln(2)-1 < ln(3)
>>>
>>>  E o produto pedido inicialmente eh menor que 3
>>>
>>>
>>>
>>>
>>>
>>>
>>>
>>>
>>> Sent from my iPad
>>> > On May 26, 2017, at 9:47 PM, Douglas Oliveira de Lima <
>>> profdouglaso.del...@gmail.com> wrote:
>>> >
>>> > Como posso fazer essa daqui:
>>> >
>>> > [2^(1/2)].[3^(1/4)].[4^(1/8)].[5^(1/16)]...<3
>>> >
>>> > Grande abraço a todos
>>> >
>>> > DouglasOliveira
>>> >
>>> > --
>>> > Esta mensagem foi verificada pelo sistema de antivírus e
>>> > acredita-se estar livre de perigo.
>>>
>>> --
>>> Esta mensagem foi verificada pelo sistema de antivírus e
>>>  acredita-se estar livre de perigo.
>>>
>>>
>>> 
>>> =
>>> Instruções para entrar na lista, sair da lista e usar a lista em
>>> http://www.mat.puc-rio.br/~obmlistas/obm-l.html
>>> 
>>> =
>>>
>>
>> --
>> Esta mensagem foi verificada pelo sistema de antivírus e
>> acredita-se estar livre de perigo.
>>
>
>
> --
> Esta mensagem foi verificada pelo sistema de antivírus e
> acredita-se estar livre de perigo.
>
>
>
> --
> Esta mensagem foi verificada pelo sistema de antivírus e
> acredita-se estar livre de perigo.

-- 
Esta mensagem foi verificada pelo sistema de antiv�rus e
 acredita-se estar livre de perigo.



Re: [obm-l] Desigualdade

2017-05-28 Por tôpico Douglas Oliveira de Lima
Então amigos, eu tive uma idéia mas não estou conseguindo concluir, vamos
lá:

Montei uma sequência e fiz a_1=3, e assim
[2^(1/2)].[3^(1/4)].[4^(1/8)].[5^(1/16)]...<3
se, e somente se,
[3^(1/4)].[4^(1/8)].[5^(1/16)]...<(3^2)/2, portanto a_2=(3^2)/2, e assim
sucessivamente escrevi a sequência
a_n=(a_(n-1)^2)/n, e usei um "leminha" para resolver, que é (a_n)^2>n+1, e
assim sai fácil, só não consegui escrever
a prova desse lema.

Mas com ele sai bem facil, pois se  (a_n)^2>n+1, então (a_n)>(n+1)^(1/2),
logo  (a_(n-1)^2)/n>(n+1)^(1/2), ou seja,
a_(n-1)>n^(1/2)(n+1)^(1/4), .,
a_1>[2^(1/2)].[3^(1/4)].[4^(1/8)].[5^(1/16)]...,
e como a_1=3, está provado.

Peço a ajuda de vocÊs para provar o lema.

 Lema:
Considere a sequência a_n=(a_(n-1)^2)/n, onde a_1=3 então (a_n)^2>n+1.



Douglas Oliveira




Em 27 de maio de 2017 18:10, Esdras Muniz <esdrasmunizm...@gmail.com>
escreveu:

> Solução muito boa.
>
> Em 27 de mai de 2017 00:37, "Gabriel Tostes" <gtos...@icloud.com>
> escreveu:
>
>> Tira ln, esse produto vai ser:
>> Sum{n>=1} ln(n+1)/(2^n) = M
>>
>> Bora escrever M de outro jeito:
>>
>> M= ln(2) + [ln(3)-ln(2)]/2 + [ln(4)-ln(3)]/2^2 + ...
>>
>> M= Sum{n>=1} (ln(n+1)-ln(n))/2^(n-1)
>>
>> Como ln(n+1)-ln(n)=ln(1+1/n)<1/n
>>
>> M=2} 1/n.2^(n-1) = L + ln(2)
>>
>> Para achar L considere:
>> 1/(1-x)= 1+x^2+x^3+...
>>
>> Integrando essa expressao temos que -(1/x).ln(1-x)= 1+x/2+x^2/3+...
>> Substituindo x=1/2 achamos que L=2ln(2)-1
>> E entao
>> M< 3ln(2)-1 < ln(3)
>>
>>  E o produto pedido inicialmente eh menor que 3
>>
>>
>>
>>
>>
>>
>>
>>
>> Sent from my iPad
>> > On May 26, 2017, at 9:47 PM, Douglas Oliveira de Lima <
>> profdouglaso.del...@gmail.com> wrote:
>> >
>> > Como posso fazer essa daqui:
>> >
>> > [2^(1/2)].[3^(1/4)].[4^(1/8)].[5^(1/16)]...<3
>> >
>> > Grande abraço a todos
>> >
>> > DouglasOliveira
>> >
>> > --
>> > Esta mensagem foi verificada pelo sistema de antivírus e
>> > acredita-se estar livre de perigo.
>>
>> --
>> Esta mensagem foi verificada pelo sistema de antivírus e
>>  acredita-se estar livre de perigo.
>>
>>
>> =
>> Instruções para entrar na lista, sair da lista e usar a lista em
>> http://www.mat.puc-rio.br/~obmlistas/obm-l.html
>> =
>>
>
> --
> Esta mensagem foi verificada pelo sistema de antivírus e
> acredita-se estar livre de perigo.
>

-- 
Esta mensagem foi verificada pelo sistema de antiv�rus e
 acredita-se estar livre de perigo.



[obm-l] Re: [obm-l] Polinômios

2017-05-27 Por tôpico Douglas Oliveira de Lima
Então:

*Podemos usar o seguinte teorema: Na divisão de um polinômio p(x) por h1(x)
o resto é r1(x); na divisão de p(x) por h2(x) o resto é r2(x); na divisão
de p(x) por h1(x).h2(x) o resto é r(x). Se r(x) é dividido por h1(x) o
resto é r1(x) e dividido por h2(x) o resto é r2(x).*

*O resto da divisão de P(x) por x4 + x2 + 1  possui de grau menor ou igual
a 3:   r(x) = ax3 + bx2 + cx + d*

*De acordo com o teorema,  ax3 + bx2 + cx + d  dividido por  x2 + x + 1
deixa resto – x + 1  e  dividido por x2 – x + 1  deixa resto 3x + 5.
Então:   i)  ax3 + bx2 + cx + d = (x2 + x + 1)(ax + e) – x + 1   =>*

*ax3 + bx2 + cx + d = ax3 + (a + e)x2 + (a + e – 1)x + e + 1*

*ii) ax3 + bx2 + cx + d = (x2 – x + 1)(ax + f) + 3x + 5   =>*

*ax3 + bx2 + cx + d = ax3 + (f – a)x2 + (a – f + 3)x + f + 5*

*\**e + 1 = f + 5   =>   e – f = 4 **\**a + e – 1 = a – f + 3   =>   e
+ f = 4   =>  e = 4   e   f = 0*

*\**d = e + 1   =>   d = 5 **\**a + e = f – a   =>   2a = – 4   =>   a
= – 2 **\**b = f – a   =>   b = 2*

*\**c = a + e – 1 = – 2 + 4 – 1   =>   c = 1**\**Ou seja:   r(x) =
– 2x3 + 2x2 + x + 5*


*Observação: O que fiz nada mais foi do que congruência aplicada a
polinômios.*


*Abraços *

*Douglas Oliveira*

Em 27 de maio de 2017 11:17, Vanderlei Nemitz <vanderma...@gmail.com>
escreveu:

> Bom dia!
>
> Alguém poderia dar uma ideia na seguinte questão? Já tentes algumas
> estratégias, mas sem êxito.
>
> Um polinômio P(x) dividido por x^2 + x + 1 dá resto -x + 1 e dividido por
> x^2 -x + 1 dá resto 3x + 5. Qual o resto da divisão de P(x) por x^4 + x^2 +
> 1?
>
> A resposta que tenho é -2x^3 + 2x^2 + x + 5.
>
> Obrigado!
>
> Vanderlei
>
> --
> Esta mensagem foi verificada pelo sistema de antivírus e
> acredita-se estar livre de perigo.

-- 
Esta mensagem foi verificada pelo sistema de antiv�rus e
 acredita-se estar livre de perigo.



[obm-l] Desigualdade

2017-05-26 Por tôpico Douglas Oliveira de Lima
Como posso fazer essa daqui:

[2^(1/2)].[3^(1/4)].[4^(1/8)].[5^(1/16)]...<3

Grande abraço a todos

DouglasOliveira

-- 
Esta mensagem foi verificada pelo sistema de antiv�rus e
 acredita-se estar livre de perigo.



Re: [obm-l] desigualdade

2017-04-30 Por tôpico Douglas Oliveira de Lima
Observe quando x=2, y=3 e z=1 a desigualdade não funciona, logo não basta
substituir x+y=a,
x+z=b e y+z=c, na verdade acho que  funciona ao "contrário" x/(x+y) + y/
(y+z) + z/(z+x) <= 2.
A não ser que seja outra questão como por exemplo:
(x+y)/z +(x+z)/y +(y+z)/x >=6 o que daria certo.

Grande abraço

Douglas Oliveira.

Em 30 de abril de 2017 10:46, marcone augusto araújo borges <
marconeborge...@hotmail.com> escreveu:

> Se x, y, z são números positivos, prove que x/(x+y) + y/ (y+z) + z/(z+x) >
> = 2
>
> --
> Esta mensagem foi verificada pelo sistema de antivírus e
> acredita-se estar livre de perigo.
>

-- 
Esta mensagem foi verificada pelo sistema de antiv�rus e
 acredita-se estar livre de perigo.



Re: [obm-l] Divisores da forma 6k + 4

2017-03-19 Por tôpico Douglas Oliveira de Lima
Então, vou tentar por um caminho aqui, qualquer coisa me corrija se faltar
algum caso:

Como 88^10=2^30.11^10, então o divisor deve asumir a forma 2^a.11^b,
portanto temos
os casos a analisar:
1) O caso e que 6q+4 é da forma 2^t, com 2<=t<=30, 6q+4=2^t, assim
3q+2=2^(t-1),
3q=2^(t-1)-2, logo 2^(t-1)=2 mod(3) ou ainda 2^t=1 mod(3), portanto t é
par, logo t=2k.
Assim de 1 a 30 temos 15 números pares.

2) O Caso e que 6q+4 é da forma 11^t, mas nsse caso não temos soluções.

3) O caso em que 6q+4=2^a.11^b, com 1<=a<=10 e 1<=b<=30, portanto
3q+2=2^(a-1).11^b,
 se a=1 então b=1 e com a>=2 teremos 3q=2^(a-1).11^b-2, então
2^(a-2).11^b=1 mod(3),
logo 2^(a+b-2)=1 mod(3), assim a+b deve ser par, o que nos dá a e b com a
mesma paridade,
e portanto temos os casos a par e b par ou a ímpar e b ímpar, ou seja
5.15+5.15=150.

Juntando os casos (1) e (2) teremos 165 divisres.

Um abraço
Douglas Oliveira.

Em 18 de março de 2017 22:16, marcone augusto araújo borges <
marconeborge...@hotmail.com> escreveu:

> Quantos divisores de 88^10 deixam resto 4 quando divididos por 6?
>
> --
> Esta mensagem foi verificada pelo sistema de antivírus e
> acredita-se estar livre de perigo.
>

-- 
Esta mensagem foi verificada pelo sistema de antiv�rus e
 acredita-se estar livre de perigo.



[obm-l] Re: [obm-l] Produto de potências(contagem)

2017-03-18 Por tôpico Douglas Oliveira de Lima
Acho que raciocínio é um pouco parecido, digamos que os expoentes dos setes
sejam a,b e c assim 7^x.7^y.7^z=7^39, logo queremos as soluções naturais dá
equação x+y+z=39 com x,y e z maiores do que ou iguais a 1 , faremos a
substituição x=a+1, y=b+1 e z=c+1 , assim a+b+c=36, portanto 38!/36!2!
=19.37=703.

Desculpe os erros , digitei do celular.
Um abraço
Douglas Oliveira.

Em 18 de mar de 2017 10:01 AM, "marcone augusto araújo borges" <
marconeborge...@hotmail.com> escreveu:

> Quantas ternas ordenadas de naturais (a,b,c) maiores que 1 são tais que
> a.b.c = 7^39?
>
> --
> Esta mensagem foi verificada pelo sistema de antivírus e
> acredita-se estar livre de perigo.
>

-- 
Esta mensagem foi verificada pelo sistema de antiv�rus e
 acredita-se estar livre de perigo.



Re: [obm-l] Um produto de 3 naturais(contagem)

2017-03-18 Por tôpico Douglas Oliveira de Lima
Então, vamos lá, eu tentei dá seguinte forma:

Fatorando o número teremos 2310=2.3.5.7.11
Logo cada número possui três possibilidades para ser "encaixado"( em a, b
ou c), desta forma teriamos 3^5 porém contamos também com números dá forma
(1,1,2310), (1,2310,1), e (2310,1,1) logo teremos 243-3=240 que dividido
por 3! Para retirar as permutas nos dá como resposta 40.

Abraços
Douglas Oliveira.

Em 18 de mar de 2017 9:58 AM, "marcone augusto araújo borges" <
marconeborge...@hotmail.com> escreveu:

> Para quantos conjuntos de inteiros positivos {a,b,c} é verdade que a.b.c =
> 2310?
>
> Alguém resolveria?Agradeço.
>
> --
> Esta mensagem foi verificada pelo sistema de antivírus e
> acredita-se estar livre de perigo.
>

-- 
Esta mensagem foi verificada pelo sistema de antiv�rus e
 acredita-se estar livre de perigo.



[obm-l] Re: [obm-l] Re: [obm-l] Re: [obm-l] Re: [obm-l] Estou tentando e não sai

2017-03-10 Por tôpico Douglas Oliveira de Lima
Olá , amigos , já tinha feito esse problema e cai na mesma duvida, se o
3,4,5 é único.
Caiu uma questão parecida no nível 2 terceira fase dá OBM que pede para
encontrar o triângulo de área mínima que possui lados inteiros e área
inteira.

Bom em relação a este problema temos como resolve-lo pelas soluções
parametricas de Brahmagupta.

Onde a=n(m^2+k^2), b=m(n^2+k^2) ,
c=(m+n)(mn-k^2) , p=mn(m+n) e
 S=mnk(m+n)(mn-k^2) onde (m,n,k)=1, mn>k^2>(m^2)n/2m+n e m>=n>=1.

Assim o que fiz foi até fácil, como o raio e igual a 1 então , S=p , assim
1=k(mn-k^2), logo só teremos k=1 e mn=2. Assim o único triângulo será o
3,4,5.

Abraços
Douglas Oliveira.


Em 8 de mar de 2017 8:22 PM, "Pedro José" <petroc...@gmail.com> escreveu:

> Boa tarde!
>
> Novamente, sem perda de generalidade, pois ao final haverá as permutações,
> a>b>c
>
> 4(a+b+c) = a^2b+a^2c+ab^2+ac^2+b^2c+bc^2-2abc-a^3-b^3-c^3
>
> f(a,b,c) = 4(a+b+c) e g(a,b,c) = a^2b+a^2c+ab^2+ac^2+b^2c+bc^2-
> 2abc-a^3-b^3-c^3
>
> a derivada parcial de g em relação a "a" é: -3a^2 + 2(c + b)a + (c -b)^2
>
> que tem duas raízes distintas de sinais contrários, já que c>b  e a é
> positivo.
>
> então o comportamento da função é:
>
> decrescente para a  crescente para  r1 < a < r2 >0
> decrescente para a > r1.
>
> mas como a > 0  para um intervalo [ao,a1], temos as seguintes
> possibilidades:
>
> (i) monótona crescente: a1 < r2
> (ii) mónotona decrescente ao > r2
> (ii) crescente de ao a r2 e decrescente de r2 a a1, ao  r2.
>
> Como f(a,b,c) é monótona crescente em relação a "a" para todo a, e uma vez
> que g(a,b,c) cresce mais rápido que f(a,b,c) com relação a "a", se fixarmos
> b e c,
> só precisamos nos preocupar com os extremos, se f(ao,b,c) < g(ao,b,c) e
> f(a1,b,c) < g(a1,b,c) não existe solução para a pertencente a [ao,a1] e b,c
> fixos.
>
> Como o triângulo é escaleno e assumimos a>b>c,
> temos que para c e b fixos, a pode variar de b+1 a c + b - 1, portanto ao
> = b+1 e a1 = c + b - 1.
>
> Para ao temos: f(ao,b,c) = 8b + 4c + 4 e g(ao,b,c) = 2(c^2-1)b +c^2 + c -
> c^3.
>
> Como b>c temos que: g(ao,b,c) > c^3 + c^2b - 2b +c^2 + c -c^3 = c^2b -
> 2b +c^2 + c
>
> Para c>= 4 temos que g (ao,b,c) > 14 b +20 >  f(ao,b,c)
>
> f(a1,b,c) = 8b + 8c + 4
>
> g(a1,b,c) = (4c+4)b -2b +1
>
> c>= 4 ==> g(a1,b,c) > 18b +1 > 10b + 8c +1 > 8b + 8c +4.
>
> Logo c<=3.
>
> c=1, não tem solução. não há triângulos escalenos com lados inteiros sendo
> um deles igual a 1. fere a desigualdade de existência  do triângulo.
>
> c=2, só há as seguintes opções b e a = b+1.
>
> temos 6b -3 = 8b + 12 ==> não há solução para c=2.
>
> para c= 3 temos duas possibilidades c= 3 e b e a= b+1 ou c =2, b e a = b+2
>
> a= b+1, b e c = 3 ==> 8b =32 ==> a=5, b=4 e c=3.
>
> a = b+ 2, b e c = 3 ==> 2b = 25, não há solução inteira.
>
> A solução é qualquer triângulo congruente a um triângulo de lados 3, 4 e
> 5. (o triângulo pitagórico de razão da PA =1).
>
> Tentei aplicar Tartaglia e garantir que a raiz fosse inteira, mas começou
> a complicar... Tive de apelar para a derivada direcional.
> Se encontrarem uma desigualdade para garantir a solução, favor postar.
>
> Saudações,
> PJMS
>
>
>
>
> Em 8 de março de 2017 07:59, Anderson Torres <torres.anderson...@gmail.com
> > escreveu:
>
>> Eu consegui algo que pode ajudar.
>>
>> [p(p-a)(p-b)(p-c)]^1/2 = p.r = p
>>
>> p^2 = p(p-a)(p-b)(p-c)
>>
>> p = (p-a)(p-b)(p-c)
>>
>> 8p = 2(p-a) * 2(p-b) * 2(p-c)
>>
>> 4(a+b+c) = (-a+b+c) * (a-b+c) * (a+b-c)
>>
>> Escreve A = (-a+b+c), B = (a-b+c), C = (a+b-c), assim A+B+C=a+b+c, e
>>
>> ABC = 4 (A+B+C)
>>
>> Isso dá para ir limitando com desigualdades e recorrer a tentativa e erro.
>>
>> 1/4 =  1/(AB) + 1/(AC) + 1/(BC)
>>
>> Em 6 de março de 2017 20:08, Pedro José <petroc...@gmail.com> escreveu:
>> > permutações e não combinações.
>> >
>> > Em 6 de março de 2017 20:06, Pedro José <petroc...@gmail.com> escreveu:
>> >>
>> >>
>> >> Boa noite!
>> >>
>> >> Fui por aí e achei:
>> >>
>> >> 4(a+b+c) = a^2b+a^2c+ab^2+ac^2+b^2c+bc^2-2abc-a^3-b^3-c^3
>> >>
>> >> Se for triângulo equilátero.
>> >>
>> >> a=b=c ==> 12a = a^3 ==> a=b=c=raiz(12), que não é inteiro.
>> >>
>> >> Se for isósceles com a<>b=c, sem perda de generalidade, pois a equação
>> é
>> >> simétrica em a,b,c.
>> >>
>> 

[obm-l] Re: [obm-l] Cálculo de determinante

2017-03-01 Por tôpico Douglas Oliveira de Lima
Muito obrigado Luís, de verdade.
Analisarei os passos, inicialmente encontrei esse determinante num livro "
Excursions in calculus" do Robert M.Young e a referência dele me levou a
procurar num livro de programação " the art of computer programming" volume
2 [263] 316.

Grande abraço
Douglas Oliveira.

Em 1 de mar de 2017 9:14 AM, "Luís Lopes" <qed_te...@hotmail.com> escreveu:

> Já mandei 2 ou 3 vezes esta mensagem para a lista.
> Não sei por que ela não aparece. Tento novamente.
>
> ===
> Oi, oi Douglas,
>
> Sauda,c~oes,
>
> Achei este problema legal e fiz uma busca por
> "determinant of gcd matrix" no google.
>
> Escolhi o link
>
> http://math.stackexchange.com/questions/126/determinant-
> value-of-a-square-matrix-whose-each-entry-is-the-g-c-d-of-row-and-c
>
> que me levou a
>
> http://waset.org/publications/9996770/two-different-
> computing-methods-of-the-smith-arithmetic-determinant
>
>
> < Obs: O resultado é MT bonito, uma potência de 2.
> Verdade para n=1,2,….6. Fura para n=7.
>
> Abs,
> Luís
>
> --
> Esta mensagem foi verificada pelo sistema de antivírus e
> acredita-se estar livre de perigo.
>

-- 
Esta mensagem foi verificada pelo sistema de antiv�rus e
 acredita-se estar livre de perigo.



[obm-l] Re: [obm-l] Re: [obm-l] Cálculo de determinante.

2017-02-28 Por tôpico Douglas Oliveira de Lima
Realmente não da uma potência de 2, mas o que dá? Qual Eureka eu encontro?

Abraço do Douglas

Em 27 de fev de 2017 8:10 PM, "Anderson Torres" <
torres.anderson...@gmail.com> escreveu:

> Isso já foi respondido em uma Eureka!
> E do que me lembre, não era uma potência de dois não.
>
> Em 22 de fevereiro de 2017 23:34, Douglas Oliveira de Lima
> <profdouglaso.del...@gmail.com> escreveu:
> > Olá caros amigos não consegui pensar no seguinte problema:
> >
> > 1) Calcular o determinante de uma matriz quadrada de ordem n , onde cada
> > elemento é o MDC entre i e j.
> >
> > Obs: O resultado é MT bonito, uma potência de 2.
> >
> > Agradeço a ajuda.
> >
> > Douglas Oliveira.
> >
> >
> > --
> > Esta mensagem foi verificada pelo sistema de antivírus e
> > acredita-se estar livre de perigo.
>
> --
> Esta mensagem foi verificada pelo sistema de antivírus e
>  acredita-se estar livre de perigo.
>
>
> =
> Instru�ões para entrar na lista, sair da lista e usar a lista em
> http://www.mat.puc-rio.br/~obmlistas/obm-l.html
> =
>

-- 
Esta mensagem foi verificada pelo sistema de antiv�rus e
 acredita-se estar livre de perigo.



[obm-l] Cálculo de determinante.

2017-02-22 Por tôpico Douglas Oliveira de Lima
Olá caros amigos não consegui pensar no seguinte problema:

1) Calcular o determinante de uma matriz quadrada de ordem n , onde cada
elemento é o MDC entre i e j.

Obs: O resultado é MT bonito, uma potência de 2.

Agradeço a ajuda.

Douglas Oliveira.

-- 
Esta mensagem foi verificada pelo sistema de antiv�rus e
 acredita-se estar livre de perigo.



[obm-l] Re: [obm-l] Re: [obm-l] Intervalos de crescimento de uma função.

2017-02-22 Por tôpico Douglas Oliveira de Lima
A dúvida é essa mesmo Ralph , questão de vestibular, segunda fase dá UFF.

Em 22 de fev de 2017 7:51 PM, "Ralph Teixeira" <ralp...@gmail.com> escreveu:

> Bom, nao tem dados na sua pergunta, mas concordo que muita gente
> confunde as frases:
>
> 1. "A funcao eh crescente em [-4,-3] e em [2,3]."
> e
> 2. "A funcao eh crescente em [-4,-3]U[2,3]."
>
> que, na minha modesta opiniao, NAO significam a mesma coisa... Pode
> ser que a primeira seja verdadeira mas a segunda seja falsa. Eh isso
> que voce estah dizendo, neh?
>
> Eh uma prova de disciplina, de vestibular ou...?
>
> (Serah que fui eu que escrevi o gabarito? :O )
>
> Abraco, Ralph.
>
> 2017-02-22 7:14 GMT-03:00 Douglas Oliveira de Lima
> <profdouglaso.del...@gmail.com>:
> > Olá caros amigos, tenho uma dúvida com relaçao ao intervalo de
> crescimento
> > de uma função.
> > Peguei uma questão da prova da UFF RJ acho que de 2008 que afirma que
> > conjunto onde a função f é crescente é C=[-4,-3]U[2,3]. Eu vejo como item
> > errado pois pela definição de função 2>-3 e
> >  f(2)<f(-3) o que contraria a definição de função crescente.
> > Gostaria de uma breve opinião a respeito.
> >
> > Agradeço a ajuda.
> > Douglas Oliveira.
> >
> > --
> > Esta mensagem foi verificada pelo sistema de antivírus e
> > acredita-se estar livre de perigo.
>
> --
> Esta mensagem foi verificada pelo sistema de antivírus e
>  acredita-se estar livre de perigo.
>
>
> =
> Instru�ões para entrar na lista, sair da lista e usar a lista em
> http://www.mat.puc-rio.br/~obmlistas/obm-l.html
> =
>

-- 
Esta mensagem foi verificada pelo sistema de antiv�rus e
 acredita-se estar livre de perigo.



  1   2   3   4   >